Módulo 3 Med. Interna Flashcards

1
Q

A 60-year-old man comes to his physician with complaints of easy fatigability and palpitations for the past 6 months. Physical examination is remarkable for pallor of skin and mucous membranes. No evidence of cardiac or respiratory disease is found. Hematologic studies show:

  • Hemoglobin: 8.4 g/dL
  • Mean corpuscular volume (MCV): 75 fl
  • Leukocyte count 9000/mm3
  • Platelet count 380,000/mm3

Serum chemistry studies show a ferritin of 25 ng/L and serum bilirubin within normal values. Peripheral blood smear shows small erythrocytes with marked variability in size. Which of the following is the most appropriate next step in management?

(A) Bone marrow biopsy
(B) Coombs test for anti-red blood cell antibodies
(C) Hemoglobin electrophoresis
(D) Test for occult blood in the stool
(E) Therapeutic trial with oral ferrous sulfate
(F) Treatment with vitamin B12 and folic acid

A

Respuesta: B

The correct answer is B. This infant has the classic presentation of pyloric stenosis. It occurs more commonly among first-born males, particularly white males. The cause of pyloric stenosis is unknown. It usually presents with nonbilious vomiting at 3-6 weeks of age. It progresses from intermittent vomiting to increasing numbers of episodes with more and more forceful vomiting. The infant acts hungry afterward, but as the vomiting continues there is loss of fluids and electrolytes resulting in hypochloremic metabolic alkalosis. Abdominal examination usually reveals a pyloric lump that feels like an olive-size mass. Treatment for pyloric stenosis is surgical; therefore, immediate consultation with the surgeon is necessary. The surgeon will preoperatively correct the metabolic alkalosis and surgically perform a pyloromyotomy. Changing the feedings to clear liquids and a solution, such as

Pedialyte (choice A), will not relieve this obstruction and will only delay the needed surgical treatment.

Obtaining abdominal x-ray films (choice C) will not delineate the specific problem. Appropriate x-ray films in this situation include either a barium swallow, which will reveal a large stomach with only thin streaks of barium in the pylorus due to the hypertrophied pylorus, or an ultrasound, which will reveal a doughnut appearance of the pyloric area.

Insertion of a nasogastric tube (choice D) will relieve an overdistended stomach, but will not treat the underlying problem.

Treating the infant with parenteral antibiotics (choice E) is not indicated. This is not an infectious condition, and prophylactic antibiotics are not used in conjunction with the surgery.

How well did you know this?
1
Not at all
2
3
4
5
Perfectly
2
Q

A 28-year-old woman presents with painful swelling of her right hand and fingers of 2 days’ duration. She has a low-grade fever and is currently menstruating. She denies a past history of sexually transmitted diseases. On examination, her temperature is 38.6 C
(101.4 F), blood pressure is 130/70 mm Hg, pulse is 110/min, and respirations are 20/min. The digits on her right hand are swollen and held in mild flexion, with papules and vesicles in the web spaces. Her left knee and ankle are swollen and tender to touch. Laboratory evaluation shows:

  • Leukocytes 12,000 with 86% neutrophils
  • Hemoglobin 14.0 g/dL
  • Platelets 220,000/mm3
  • Erythrocyte sedimentation rate 43 mm/h

X-ray films of the hand, knee, and ankle show no evidence of fracture. Which of the following tests is most likely to confirm the likely diagnosis?

(A) Cultures of cervix, rectum, throat, and blood
(B) Blood cultures
(C) Arthrocentesis for bacterial cultures
(D) Synovial fluid analysis for cell count
(E) Synovial fluid Gram stain

A

Respuesta: D

The correct answer is D. The condition is called transient hypogammaglobulinemia of infancy. It is due to exhaustion of maternally supplied IgG (the only antibody to cross the placenta in significant amounts) before the infant has begun to produce significant amounts of his own antibodies. Affected infants typically go through several months to years of being very vulnerable to infection, and then improve as their immune systems mature and they are able to produce more IgG. The condition does not present in the neonatal period because maternal IgG can cross the placenta during the third trimester of intrauterine life. The maternally supplied IgG is usually exhausted by about the 4th to 6th month of life.

IgA (choice A) is found in secretions and in serum but does not cross the placenta.

IgD (choice B) is a transiently produced antibody during B cell development that is present in trace amounts in serum.

IgE (choice C) is the antibody associated with allergic reactions; it does not cross the placenta.

IgM (choice E) is a large antibody found in serum that does not cross the placenta.

How well did you know this?
1
Not at all
2
3
4
5
Perfectly
3
Q

A 39-year-old California rancher consults a physician because of
chronic abdominal pain. The man has been a sheepherder for 23 years. Physical examination is notable for a palpable liver mass but is otherwise unremarkable. Ultrasound demonstrates a 15-cm cyst bearing multiple daughter cysts in the liver. CT confirms the presence of the cysts and demonstrates the presence of a finely calcified cyst rim. Which of the following is the most likely diagnosis?

(A) Ascariasis
(B) Echinococcosis
(C) Fascioliasis
(D) Schistosomiasis
(E) Toxocariasis

A

Respuesta: B

The correct answer is B. This is hydatid disease, due to infection with E. granulosus (rarely Echinococcus multilocularis). The life cycle of this parasitic worm usually alternates between sheep and canine carnivores (including sheep herding dogs). Man is an accidental host. Endemic areas correspond to the major sheepherding areas of the world: the Mediterranean, the Middle East, Australia, New Zealand, South Africa, and South America. Smaller foci of the disease are found in California, Canada, and Alaska. The ingested egg hatches in the intestine, and the larva migrates to the human liver, lungs, or, less commonly, other body sites. Over years, the larva forms the hydatid cyst, which is a large, fluid-filled bladder that develops multiple brood capsules in its periphery, each of which contains numerous small infective scolices. So long as the cyst does not rupture, the patient may be asymptomatic. However, rupture (including accidental surgical rupture) can cause an anaphylactic reaction (the cyst fluid is highly antigenic) or a “metastatic” infection, as the up to millions of infectious scolices are released. Treatment can be either with very careful surgical resection or with percutaneous aspiration under CT guidance followed by instillation of a scolecoidal agent and then reaspiration. If the case is inoperable, or if an intraoperative spillage occurs, albendazole can be used to suppress growth or kill the cysts.

Ascariasis (choice A) can cause biliary obstruction by the adult worms and granulomas in the liver by the larvae.

Fascioliasis (choice C) can acutely cause tender hepatomegaly with fever and eosinophilia, and can chronically cause cholangitis and biliary fibrosis.

Schistosomiasis (choice D) can cause a severe granulomatous reaction to the ova, producing hepatosplenomegaly, pipestem fibrosis, and portal hypertension.

Toxocariasis (choice E) can cause visceral larva migrans and hepatomegaly with granulomas.

How well did you know this?
1
Not at all
2
3
4
5
Perfectly
4
Q

A solitary nodule is detected on a chest x-ray film in an otherwise healthy 25-year-old man. The patient has been smoking 10 cigarettes daily for 3 years. The nodule is located in the right middle lobe and measures approximately 1.5 cm. Previous chest x-ray films are not available for comparison. CT scan reveals a solitary lung nodule with a smooth contour and diffuse calcifications. No other pulmonary lesions are found. Physical examination and routine laboratory tests are normal. Which of the following is the most likely diagnosis?

(A) Aspergilloma
(B) Bronchogenic carcinoma
(C) Hamartoma
(D) Pulmonary abscess
(E) Sarcoidosis
(F) Secondary (reactivated) tuberculosis

A

Respuesta:

The correct answer is C. This case raises the problem of the clinical approach to a solitary pulmonary nodule radiographically detected in an otherwise healthy subject. In large surveys, 60% of solitary pulmonary nodules are benign, and granulomas represent the most common benign lesion. However, there is no infallible clinical or radiologic set of criteria that can discriminate between benign and malignant lesions. Factors favoring a benign lesion include young age (< 40 to 45 years), small size (< 2 cm) and smooth margins of the lesion, absence of symptoms, and slow growth on successive films. Generally, calcification is not a malignant feature, and presence of “popcorn-like” calcifications definitely favors hamartoma. A hamartoma is a malformative lesion resulting from random admixture of tissues normally present in the lung, including cartilage, bronchial mucosa, and smooth muscle. It is usually discovered incidentally.

An aspergilloma (choice A) is a fungus ball (mycetoma) that develops in a pre-existing lung cavity. It may be seen as an asymptomatic radiographic abnormality (a crescent of air outlining a solid mass that moves with position changes) that is usually in the upper lobe. An aspergilloma can cause hemoptysis, and there are often other systemic signs. Hemoptysis is frequently present, and signs and symptoms of the underlying disease should be found.

Bronchogenic carcinoma (choice B) may present as a solitary nodule, but patients are typically older and have a much longer exposure to cigarette smoking. Calcifications usually are not seen in malignant pulmonary tumors.

Pulmonary abscess (choice D) manifests as a pulmonary infiltrate with cavitation and, frequently, an air-fluid level. Accompanying symptomatology, such as fever and cough, is usually present.

Sarcoidosis (choice E) affects the lungs frequently. A diffuse multinodular infiltrate is seen on chest x-ray, most commonly associated with hilar lymphadenopathy. The patient may have signs and symptoms of restrictive pulmonary disease.

Secondary (reactivated) tuberculosis (choice F) manifests with multiple nodular and cavitary infiltrates in the upper lobes; the patient has low-grade fever, malaise, weight loss, and cough. However, primary tuberculosis may result in a calcified nodule within the lung parenchyma, which is the remnant of an old Ghon complex.

How well did you know this?
1
Not at all
2
3
4
5
Perfectly
5
Q

A 65-year-old woman is admitted with a 3-week history of headache over the right temporal region, malaise, fever, morning stiffness, and weight loss. On physical examination, scalp tenderness is appreciated. Her temperature is 38.5 C (101.3 F), blood pressure is 142/84 mm Hg, pulse is 85/min, and respirations are 14/min. There is no loss of visual acuity, and funduscopic examination is unremarkable. Laboratory studies show the following:

  • Hematocrit: 40.3%
  • Hemoglobin: 11.9 g/dL
  • Leukocytes: 7800/mL (neutrophils 68%)
  • Erythrocyte sedimentation rate (ESR): 80 mm/h

After reviewing the results, the physician initiates highdose prednisone therapy. Which of the following is most likely to confirm the diagnosis?

(A) CT scan of the head
(B) Lumbar puncture
(C) Muscle biopsy
(D) Temporal artery biopsy
(E) Visual evoked potentials

A

Respuesta: D

The correct answer is D. The patient has signs, symptoms, and laboratory findings consistent with giant cell arteritis, which is a systemic disease overlapping with polymyalgia rheumatica. It affects the superficial temporal artery most commonly but may involve any medium-sized or large caliber artery in the body. The most characteristic elements in the diagnosis include scalp tenderness (sometimes associated with palpation of a nodular and tender temporal artery), elevated ESR, a frequently normal leukocyte count, and age older than 55 years. The major risk of this condition is blindness deriving from extension of the inflammatory process to the ophthalmic artery. Prednisone therapy must be immediately started, and a biopsy of the temporal artery should be obtained to confirm the diagnosis.

CT scan of the head (choice A) would be useful to obtain information in case of suspected intracranial lesions, especially bleeding, infarct, or a space-occupying mass.

Lumbar puncture (choice B) for CSF examination would have no diagnostic value in this patient. Meningitis would develop more rapidly, usually with nuchal headache and rigidity. Subarachnoid hemorrhage results in thunderclap headache followed by changes in mental status.

Muscle biopsy (choice C) is useful in investigating muscular disorders. Myalgia, arthralgia, and stiffness in the pelvic and shoulder girdles are often present in this condition, which may mimic a myopathy.

Visual evoked potentials (choice E) are especially useful in evaluation of optic nerve involvement in demyelinating diseases.

How well did you know this?
1
Not at all
2
3
4
5
Perfectly
6
Q

A 35-year-old man goes to the emergency department because he has developed a severe case of hives. He has never had hives before. Physical examination demonstrates multiple wheals and
erythematous patches over his body, which respond to subcutaneous epinephrine. The patient’s sclera are slightly yellow-tinged. Results of the chest and abdomen examination are within normal limits. Screening biochemistry tests demonstrate an AST (SGOT) of 350 U/L and an ALT (SGPT) of 300 U/L. Which of the following is the
most likely diagnosis?

(A) Alcoholic cirrhosis
(B) Alpha1-antitrypsin deficiency
(C) Hemochromatosis
(D) Hepatitis A
(E) Hepatitis B

A

Respuesta: E

The correct answer is E. Urticaria, or hives, is a common condition that can be quite difficult to manage because it may have such a wide variety of triggers. The underlying basis of the condition is mediator- (histamine, serotonin, leukotrienes) driven vasodilation with accompanying dermal edema. The triggers for mediator release from mast cells or basophils may be either allergic (IgE bound to antigen) or nonimmunologic direct pharmacologic effects. Although there is a wide range of possible triggers for urticaria, it is important to note that up to 25% of cases of acute hepatitis B present with urticaria. It is therefore well worth checking the sclera (the most visible site in tanned or dark-skinned individuals) for jaundice in patients with newly diagnosed hives. Other triggers include contact chemicals, drugs, food allergens, pressure, sunlight, insect stings, and hereditary predispositions.

All the other choices can produce acute or chronic hepatitis but do not have a significant association with urticaria.

How well did you know this?
1
Not at all
2
3
4
5
Perfectly
7
Q

A 47-year-old Brazilian immigrant presents with fatigue and dyspnea. He has been healthy for the past 10 years. The patient denies fever, cough, chills, or weight loss. On physical examination there are no murmurs, the pulse and the rhythm are regular, S1 is normal, and S2 is split. This split increases with inspiration and persists with expiration. Which of the following ECG findings is most consistent with the auscultatory findings in this patient?

(A) Acute ST segment elevation in the anterior leads
(B) Decreased PR interval
(C) Early repolarization
(D) Marked T wave inversion
(E) Right bundle branch block

A

Respuesta: E

The correct answer is E. A persistently wide split S2 is typically seen in patients with right bundle branch block, pulmonic stenosis, pulmonary embolus, and ectopic or pacemaker beats originating in the left ventricle. All these conditions produce delayed function of the right ventricle. In a Brazilian patient, Chagas disease (caused by Trypanosoma cruzi) should be considered as a possible cause of heart block. This trypanosomal infection is endemic in Central and South American countries, and is a cause of rhythm disturbances, cardiomyopathy, and thromboembolism.

ST segment elevation (choice A) is seen in acute anterior myocardial infarction.

A short PR interval (choice B) is associated with a loud S1 (when the mitral valve slams shut).

Early repolarization (choice C) will produce ECG changes only with no specific findings on auscultation.

Although modest T wave changes can sometimes be associated with right bundle branch block, marked T wave inversions (choice D) are more typical of right or left ventricular hypertrophy and myocardial ischemia or infarction.

How well did you know this?
1
Not at all
2
3
4
5
Perfectly
8
Q

An otherwise healthy 60-year-old man undergoes a health
maintenance examination. Physical examination and medical history are unremarkable. A blood chemistry panel is normal except for a serum calcium level of 11 mg/dL when corrected for serum albumin. The measurement is repeated two times, giving values of 10.5 mg/dL and 11.2 mg/dL, respectively. Serum phosphorus is 2.5 mg/dL, and alkaline phosphatase is 50 U/L. Immunoradiometric assay (IRMA) reveals higher than normal serum levels of parathyroid hormone. Urine calcium excretion is within normal limits. The patient denies previous renal colic or urinary tract infections. Which of the following is the most appropriate next step in management?

(A) Bone x-ray films
(B) Extensive cancer screening
(C) Generous fluid intake
(D) Treatment with bisphosphonates (e.g., alendronate)
(E) Surgical exploration of the neck

A

Respuesta: C

The correct answer is C. Hyperparathyroidism is one of the most frequent endocrinologic conditions, found in 1 in 1000 adults. In most cases, hyperparathyroidism is asymptomatic, manifesting only with hypercalcemia often discovered incidentally in the course of routine laboratory investigations conducted for other reasons. The most common cause is a parathyroid adenoma. Calcium levels should be corrected for albuminemia, since most of the calcium is bound to serum albumin. If hypercalcemia is the only clinical sign, without associated complications such as renal stones, bone disease, or cataracts, abundant fluid intake is the only measure recommended to prevent formation of calcium stones in the urinary system.

Bone x-ray films (choice A) are not necessary for the diagnostic work-up of hypercalcemia and hyperparathyroidism, unless bone pain or pathologic fractures are present. X-ray films may reveal bone rarefaction, which is often more pronounced in the phalanges and in subperiosteal locations.

Extensive cancer screening (choice B) would be appropriate if hypercalcemia were suspected to be secondary to neoplasms (e.g., lung, breast, renal cell carcinoma, multiple myeloma). Hypercalcemia of malignancy can be due to secretion of PTH-like substances or bone destruction by metastases. In either case, plasma levels of PTH detected by IRMA would be low. In fact, the PTHlike peptides produced by some tumors (lung cancer) are not identified by IRMA. Alkaline phosphatase would be high in the presence of osteolysis.

Treatment with bisphosphonates (e.g., alendronate; choice D) may serve as an alternative treatment to prevent excessive bone resorption.

Surgical exploration of the neck (choice E) is aimed at finding the source of increased PTH production, which is usually a parathyroid adenoma. Removal of the adenoma is recommended when patients have symptomatic hyperparathyroidism, with recurrent renal stones or bone disease. Indications for surgical treatment in asymptomatic patients include very high calcium levels, high urinary excretion of calcium, extreme bone loss, or difficulty in medical follow-up.

How well did you know this?
1
Not at all
2
3
4
5
Perfectly
9
Q

A 27-year-old African American man visits his primary care
physician because of recent onset of “yellowness in the white of his eyes.” His recent history is significant for a “chest cold” for which he is taking trimethoprim-sulfamethoxazole; he is also taking fluoxetine for depression. On examination, the sclera are icteric and the mucosa beneath the tongue appears yellow. No hepatosplenomegaly is present. Laboratory studies are as follows:

  • Hemoglobin 11.1 g/dL
  • Hematocrit 34%
  • Total bilirubin 6.2 mg/dL
  • Conjugated (direct) bilirubin 0.8 mg/dL
  • Alkaline phosphatase 77
  • AST (SGOT) 24
  • ALT (SGPT) 22

Which of the following is the most likely explanation for this patient’s jaundice?

(A) Acute infectious hepatitis
(B) Cholestatic liver disease
(C) Drug reaction from fluoxetine
(D) Drug reaction from trimethoprim-sulfame-thoxazole

A

Respuesta: D

The correct answer is D. This man has glucose-6-phosphate dehydrogenase (G6PD) deficiency (as do 10% of African American males). G6PD serves to protect the RBCs from oxidative damage by maintaining high intracellular levels of NADPH. People of Mediterranean descent can also have G6PD deficiency, but to a much greater degree. Therefore, hemolytic episodes in this population are more severe (and can be fatal), as compared with those in the African American population, which are usually mild and self-limited. Common oxidative stressors that initiate hemolysis are drug reactions (especially sulfa drugs), febrile illnesses, and fava bean ingestion.

Acute infectious hepatitis (choice A) would more likely present with fatigue, fever, abdominal pain, hepatomegaly, and high elevations of AST and ALT (often into the 1000s).

Cholestatic liver disease (choice B) more often presents with elevation of alkaline phosphatase, along with mild elevations of AST and ALT. This patient has elevated unconjugated bilirubin levels, as in hemolytic disorders. Both hepatocellular (hepatitis) and cholestatic liver disease cause more conjugated (as opposed to unconjugated) hyperbilirubinemia.

The most common side effects of fluoxetine (a selective serotonin reuptake inhibitor; choice C) are anxiety, agitation, and insomnia.

How well did you know this?
1
Not at all
2
3
4
5
Perfectly
10
Q

A 26-year-old woman complains of painful and frequent urination for 2 days. She has no significant medical history and has had a monogamous sexual relationship with another woman for the past year. Her temperature is 37.2 C (99 F), blood pressure is 120/70 mm Hg, pulse is 68/min, and respirations are 12/min. There is no costovertebral angle tenderness, the abdomen is soft, and there is mild suprapubic tenderness. The pelvic examination is within normal limits except for tenderness at the urethral meatus. Urinalysis reveals 23 white blood cells per high-power field. Which of the following is the most likely pathogen?

(A) Chlamydia trachomatis
(B) Escherichia coli
(C) Klebsiella pneumoniae
(D) Proteus sp.
(E) Staphylococcus saprophyticus

A

Respuesta: B

The correct answer is B. Escherichia coli, a coliform bacteria, is the most common cause of urinary tract infection (UTI) and is therefore seen much more often than Klebsiella (choice C) and Proteus (choice D). E. coli causes about 80% of UTIs in patients without urologic abnormalities. Coliform bacteria colonize the urethra, and ascending infection may lead to cystitis or pyelonephritis.

Chlamydia trachomatis (choice A) is part of the differential diagnosis for UTI in a sexually active patient with dysuria. However, the patient often has a vaginal discharge. Urine culture usually shows < 100 organisms. On physical examination, the patient with Chlamydia may have cervical motion tenderness if cervicitis is present.

Staphylococcus saprophyticus (choice E) is another common cause of UTIs in young women, often after heterosexual intercourse.

How well did you know this?
1
Not at all
2
3
4
5
Perfectly
11
Q

A 72-year-old man complains of malaise and easy fatigability for the past 3 weeks. His past medical history is significant for gout and pneumonia. He lives alone and usually drinks two six-packs of beer daily. His temperature is 36.9 C (98.4 F), blood pressure is 160/90 mm Hg, pulse is 88/min, and respirations are 19/min. Thyroid palpation is normal, and heart, lung, and abdomen examination results are within normal limits. There is a diffuse ecchymotic rash spreading out from hair follicles on the limbs and trunk. The patient most likely has a deficiency of which of the following vitamins?

(A) Niacin
(B) Thiamin
(C) Vitamin B12
(D) Vitamin C
(E) Vitamin D

A

Respuesta: D

The correct answer is D. This question examines the different aspects of vitamin deficiencies. Scurvy is a deficiency of vitamin C that may occur in older men who cook for themselves. The features are perifollicular hemorrhage and purpura, splinter hemorrhages, and gum involvement. Normochromic, normocytic anemia is common.

Niacin deficiency (choice A), known as pellagra, is a chronic wasting disease associated with dermatitis, dementia, and diarrhea. The skin lesions are characterized by hyperkeratosis, hyperpigmentation, and desquamation. The course is progressive over several years. Niacin is found in cereals.

Vitamin B1 (thiamine) deficiency (choice B), known as beriberi, occurs in alcoholics and food faddists. Two manifestations of deficiency include cardiovascular disease (high output failure) and neurologic disorders (e.g., Wernicke-Korsakoff syndrome, characterized by peripheral neuropathy, a global confusional state, retrograde amnesia, and confabulation).

Vitamin B12 deficiency (choice C) causes a macrocytic anemia. Patients may complain of a sore tongue or weight loss. Neurologic manifestations, including weakness and ataxia from demyelination, are the most worrisome. Causes include pernicious anemia, gastrectomy, and ileal abnormalities.

Vitamin D deficiency (choice E) causes disorders of bone mineralization, namely rickets in children and osteomalacia in adults.

How well did you know this?
1
Not at all
2
3
4
5
Perfectly
12
Q

A 40-year-old man with a history of type V hyperlipoproteinemia is brought to the emergency department 3 hours following the abrupt onset of severe deep epigastric pain, nausea, and vomiting. The pain is steady and radiates to the back. The patient is agitated and has cool, clammy skin. His temperature is 38.5 C (101 F), blood pressure is 100/70 mm Hg, pulse is 110/min, and respirations are 22/min. Abdominal examination reveals tenderness in the upper abdomen, without guarding. A plain x-ray film shows an air-filled intestinal loop in the left upper quadrant. Laboratory investigations:

Glucose: 150 mg/dL
LDH: 150 U/L
ALT: 90 U/L
AST: 80 U/L
Amylase: 120 U/L
Lipase: 30 U/L
Calcium: 7 mg/dL
C-reactive protein: 1.2 mg/dL

Which of the following is the most likely diagnosis?
(A) Acute cholecystitis
(B) Acute hepatitis
(C) Acute pancreatitis
(D) Bowel perforation
(E) Mesenteric ischemia
(F) Ureteral lithiasis

A

Respuesta: C

The correct answer is C. Although the whole clinical presentation is characteristic of acute pancreatitis, normal or slightly elevated serum levels of pancreatic enzymes seem to contrast with such a diagnosis. Types I and V hyperlipoproteinemia are rare conditions predisposing to acute pancreatitis (the two most frequent are cholelithiasis and alcoholism). However, hypertriglyceridemia itself may often falsely depress amylase and lipase levels in the presence of otherwise typical clinical features of pancreatitis. Note, however, other classic laboratory and radiologic signs of acute pancreatitis present in this case: neutrophilic leukocytosis, hyperglycemia, hypocalcemia, elevated C-reactive protein, and the “sentinel loop” (airfilled loop of small bowel in the left upper quadrant). The latter parameter is often used as a radiologic marker of pancreatic damage.

Acute cholecystitis (choice A) is associated with gallstones in the great majority of cases. Often, it develops when a stone occludes the cystic duct. Severe pain in the right upper quadrant or epigastrium and leukocytosis are present, but none of the other signs characteristic of acute pancreatitis are seen.

Acute hepatitis (choice B) is associated with increased levels of serum aminotransferases. Pain is relatively mild compared with the extreme pain of acute pancreatitis. Jaundice is often present, although pancreatitis may also cause mild elevation of direct bilirubin because the pancreas becomes swollen and the common bile duct is blocked.

Bowel perforation (choice D) also presents with a dramatic clinical picture of pain of sudden onset, shock, and rigid abdomen. Pancreatic enzymes may be elevated. A plain x-ray film taken in upright position reveals air underneath the diaphragm.

Mesenteric ischemia (choice E) due to cardioembolism or artherosclerosis of mesenteric vessels presents with acute abdominal pain that is typically out of proportion to physical examination findings. The typical patient is over 50 years old and presents with acute left-sided abdominal pain that begins in the left iliac fossa, with nausea, vomiting, diarrhea, and abdominal guarding. The patient may have had similar previous episodes, or there may be associated symptoms of cardiovascular disease. Plain films show thickened bowel walls, indicating a paucity of gas in the intestines. Specific radiologic signs are pneumatosis intestinalis (i.e., submucosal gas), bowel wall thumbprinting, and portal vein gas. Although patients with hypertriglyceridemia are at risk for cardiovascular diseases, the most likely cause of this patient’s symptoms is acute pancreatitis.

Ureteral lithiasis (choice F) is in the differential diagnosis of conditions mimicking acute pancreatitis. However, the pain is often referred to the flank region and radiates toward the ipsilateral perineum. Gross or microscopic hematuria is usually present. Pancreatic enzymes are not elevated, and neutrophilic leukocytosis is absent.

How well did you know this?
1
Not at all
2
3
4
5
Perfectly
13
Q

An 18-year-old man has had rhinorrhea and a sore throat for 2 days. He has no significant past medical history. His temperature is 39.3 C (102.6 F), pulse is 110/min, and respirations are 20/min. He has tender anterior and posterior cervical lymphadenopathy and an erythematous pharynx with white exudates on the tonsils. The remainder of his examination is normal. Which of the following is the most likely causal organism?

(A) Candida albicans
(A) Candida albicans
(B) Haemophilus influenzae
(C) Staphylococcus aureus
(D) Streptococcus pneumoniae
(E) Streptococcus pyogenes

A

Respuesta: E

The correct answer is E. This patient has the classic triad of fever, exudative pharyngitis, and cervical lymphadenopathy, strongly suggestive of streptococcal infection (group A beta hemolytic Streptococcus).

Candida albicans (choice A) rarely causes the above clinical scenario. It is seen commonly in immunosuppressed patients, diabetics, and those recently on antibiotics. It causes characteristic white patches of exudate on mucosa.

Haemophilus influenza (choice B), Staphylococcus aureus (choice C), and Streptococcus pneumoniae (choice D) rarely cause exudative pharyngitis.

How well did you know this?
1
Not at all
2
3
4
5
Perfectly
14
Q

Due to a recent and sudden death of a rival college football player, a local university wishes to implement changes in its health care policies. The school is particularly concerned about the risk for sudden cardiac death in players with previously undiagnosed heart conditions. A preparticipation evaluation is to be performed on all of the school’s athletes. Which of the following is considered the most cost-effective method of screening young athletes at risk for sudden cardiac death?

(A) Careful medical history and examination
(B) Chest x-ray
(C) Echocardiography
(D) Exercise electrocardiography
(E) Resting electrocardiography

A

Respuesta: A

The correct answer is A. Sudden cardiac death (SCD), defined as any unexpected death of proven cardiac origin, is a rare event in young, competitive athletes. However, it raises the question of which preparticipation screening methods should be used for prevention. A consensus document published by the American Heart Association in 1996 (Circulation 1996; 94:850) indicates that the most feasible and cost-effective approach is to perform a careful medical history (including personal and family history) and cardiac examination to identify subjects at risk. Pertinent histories, cardiac auscultation, and assessment of exercise-induced symptoms are all essential to primary prevention of SCD in the athletic population.

Chest x-ray (choice B) would not be an adequate screening test to detect cardiac abnormalities that may result in sudden death.

Echocardiography (choice C) is certainly an effective test to demonstrate cardiac abnormalities, such as left ventricular hypertrophy or valvular disease, but its cost as a preparticipation screening in all competitive athletes would be exorbitant.

Both exercise cardiography (choice D) and resting cardiography**(choice E) **would also result in excessive costs.

How well did you know this?
1
Not at all
2
3
4
5
Perfectly
15
Q

A 20-year-old male college student is participating in the New York City Marathon and collapses one third of the way through the race. He is a well developed, athletic man who frequently plays basketball and tennis. He has no past medical history, except for a tonsillectomy at 9 years of age. There were no symptoms before he collapsed to the ground and lost consciousness. The patient is immediately rushed to the nearest emergency room but is pronounced dead on arrival. Which of the following is the most likely underlying cause of his sudden death?

(A) Aortic stenosis
(B) Arrhythmogenic right ventricular dysplasia
(C) Coronary anomalies
(D) Hypertrophic cardiomyopathy
(E) Isolated left ventricular hypertrophy
(F) Myocarditis
(G) Ruptured aorta

A

Respuesta: D

The correct answer is D. A fatal arrhythmia is the immediate cause of demise in sudden cardiac death (SCD), but the underlying conditions are extremely variable. Nonatherosclerotic causes are prevalent in the young population, but coronary artery disease becomes more frequent in athletes older than 40. Hypertrophic cardiomyopathy has been found in approximately ⅓ of cases of SCD in young athletes who present with SCD. This condition is frequently hereditary (hence the need for a careful family history) and may manifest with arrhythmias, chest pain, and signs of subaortic stenosis. Coronary anomalies (choice C), including atherosclerotic changes, represent the second most frequent cause of SCD in competitive athletes.

Aortic stenosis (choice A), arrhythmogenic right ventricular dysplasia (choice B), isolated (and otherwise unexplained) left ventricular hypertrophy (choice E), myocarditis (choice F), and ruptured aorta (choice G) are rare causes of SCD. Some studies have shown that an increased myocardial mass, per se, is a risk factor for SCD, even without coexisting pathologic changes.

How well did you know this?
1
Not at all
2
3
4
5
Perfectly
16
Q

A seriously ill AIDS patient is admitted to a hospital. He has multiple infections, including Pneumocystis carinii pneumonia, pulmonary cytomegalovirus infection, and candidiasis of the esophagus and possibly other sites. Screening chemistry studies are drawn, including electrolytes. Which of the following abnormalities would be most likely seen in this setting?

(A) Hyperkalemia
(B) Hypermagnesemia
(C) Hypocalcemia
(D) Hyponatremia
(E) Hypophosphatemia

A

Respuesta: D

The correct answer is D. You should be aware that roughly half of all hospitalized AIDS patients have hyponatremia. There are many reasons for abnormal sodium metabolism in seriously ill AIDS patients, who may have multiple organ systems involved with the disease. Some of the drugs these patients receive impair renal water excretion, hypotonic fluids may be administered during the course of therapy, and they may also have impaired renal function.

Cytomegalovirus or mycobacteria may cause destructive adrenalitis, or ketoconazole may impair adrenal glucocorticoid and mineralocorticoid synthesis. Pulmonary and CNS infections may induce the syndrome of inappropriate ADH secretion (SIADH). All these causes, often more than one in an individual case, contribute to the high incidence of hyponatremia in AIDS patients. If a specific cause can be identified and ameliorated, that will often improve the hyponatremia. Otherwise, fluid restriction is used for mild abnormalities; administration of hypertonic saline can be used to (slowly) correct more severe hyponatremia.

Disorders of potassium (choice A), magnesium (choice B), calcium (choice C), and phosphate (choice E) can occur but are much less common than disorders of sodium in AIDS patients.

How well did you know this?
1
Not at all
2
3
4
5
Perfectly
17
Q

A 31-year-old man is admitted to the hospital for suspicion of gastrointestinal bleeding. He has no significant past medical history but takes daily nonsteroidal antiinflammatory agents for pain in his knee. He presented to the hospital 6 hours ago after he noticed melanotic stools while at home. He is observed to have copious bright red blood per rectum. On physical examination, he is tachycardic, and his peripheral pulses are faint but present. His mental status appears normal. His extremities are cool to the touch. An intravenous line is placed. Which of the following is the most appropriate next step in management?

(A) Order an urgent type and cross match for blood
(B) Order an urgent hematocrit level
(C) Begin parenteral administration of large volumes of normal saline solution
(D) Begin parenteral administration of large volumes of colloid solution
(E) Place two additional large bore peripheral intravenous catheters

A

Respuesta: C

The correct answer is C. The management of acute hemorrhage is the same for almost all patients, regardless of the etiology. In this case, a patient who is actively bleeding with apparent marginal vital signs requires immediate restoration of blood pressure via fluid resuscitation with at least 3 L of crystalloid solution (normal saline or lactated Ringer’s) for every liter of blood lost. Tachycardia and hypotension are signs of a moderate to severe loss of blood volume, and no delay in initiating fluid therapy is warranted.

Ordering an urgent type and cross match for blood (choice A), although appropriate for the overall short-term management of this patient, is not an acceptable therapeutic option in the face of active bleeding with no fluid resuscitation in progress.

Ordering an urgent hematocrit level (choice B) is not useful in this case because hematocrit levels do not change for at least 4 hours after an acute bleed. In addition, it offers no therapeutic benefit and will not change the short-term management of this patient.

Beginning parenteral administration of large volumes of colloid solution (choice D) is not indicated in this case. In fact, colloid (albumin, Hetastarch, Hetaspan) is rarely indicated for fluid resuscitation since it may actually precipitate pulmonary edema. The only clear indication for colloid is in the therapy of early burns, as these patients have capillary leaks and are losing protein and albumin.

Placing two additional large bore peripheral IV catheters (choice E) is indicated only AFTER fluid resuscitation has been started through whatever peripheral or central access is available. The concept here is that large bore IV catheters or central catheters are required for aggressive fluid resuscitation, but not at the expense of delaying therapy through an already available, but smaller, route.

How well did you know this?
1
Not at all
2
3
4
5
Perfectly
18
Q

A 40-year-old man presents to the physician because of exertional dyspnea of recent onset. The patient appears comfortable at rest but says that he becomes short of breath with minimal effort. His temperature is 37 C (98.6 F), blood pressure is 162/65 mm Hg, pulse is 92/min with a rapid rise and fall, and respirations are 15/min. Chest examination reveals a prominent and laterally displaced apical impulse. A soft diastolic decrescendo murmur is heard along the left sternal border. Bilateral crackles are present at the lung base. The liver is not palpable, and there is no sign of peripheral edema. Which of the following is the most likely diagnosis?

(A) Aortic insufficiency
(B) Aortic stenosis
(C) Hypertrophic obstructive cardiomyopathy
(D) Infective endocarditis
(E) Mitral stenosis
(F) Ventricular septal defect

A

Respuesta: A

The correct answer is A. The patient manifests early left ventricular failure secondary to aortic regurgitation. The diastolic murmur in decrescendo along the left sternal border and the wide differential between systolic pressure and diastolic pressure are highly characteristic. The rapid rise and fall of peripheral pulses is known as Corrigan pulse. Similar hemodynamic changes (hyperdynamic circulation) may be observed in hyperthyroidism, large arteriovenous fistulas, beriberi, and patent ductus arteriosus.

Aortic stenosis (choice B) produces a “diamond-shaped” (crescendo-decrescendo) systolic murmur often radiating to the neck. In contrast to aortic regurgitation, aortic stenosis is associated with a narrow differential between systolic and diastolic pressures.

Hypertrophic obstructive cardiomyopathy (HOCM; choice C) may simulate aortic stenosis or coronary artery disease in symptoms. HOCM often presents with symptoms of exertional angina, dyspnea, and/or syncope. It is also widely known to be a cause of sudden cardiac death in young athletes. Diagnosis is usually made by recognizing the signs associated with outflow obstruction and the symptoms that are predominantly characterized by diastolic dysfunction. Characteristic findings on physical examination include a bifid carotid pulse, an S4 heart sound, and a harsh systolic crescendo-decrescendo located at the apex and left sternal border. The murmur increases with the Valsalva maneuver upon standing and with amyl nitrite, and it decreases with sudden squatting, leg raising, and handgrip exercises.

Infective endocarditis (choice D) may be accompanied by murmurs, usually secondary to valvular insufficiency, but systemic signs and symptoms of infection would be present.

Mitral stenosis (choice E) leads to a diastolic rumbling murmur audible at the apex or central precordium. An opening snap soon after S2 often precedes the murmur and is related to the forced opening imposed by the atrial contraction on rigid mitral valve leaflets.

Ventricular septal defect (VSD; choice F), the most frequent congenital cardiac anomaly, is associated with a pansystolic murmur at the left sternal border, often accompanied by a thrill.

How well did you know this?
1
Not at all
2
3
4
5
Perfectly
19
Q

A 42-year-old man with AIDS presents with a chief complaint of
persistent watery, non-bloody diarrhea. He is not on any medications
and denies recent travel or fever. On physical examination, his
abdomen is slightly bloated, with mild tenderness to palpation.
There is no occult blood in his stool. Stool samples for leukocytes, culture, ova, and parasites are all negative × 3. Which of the
following is the most appropriate next step in diagnosis?
(A) Abdominal CT
(B) Cytomegalovirus (CMV) antigenemia
(C) Modified acid-fast stain of the stool
(D) PPD test
(E) Small bowel biopsy

A

Respuesta:

The correct answer is C. In cases of HIV and persistent diarrhea, the differential diagnosis includes Cryptosporidium and Isospora. A fresh stool specimen should be examined for parasites using a modified acid-fast stain for both pathogens. These protozoal infections are the most common enteric protozoal infections in AIDS patients throughout the world.

Abdominal CT scan (choice A) may show changes in the colon wall consistent with inflammation and edema. However, the scan will probably not be helpful in the principal diagnosis of this patient’s condition.

CMV antigenemia (choice B) is an index of infection with CMV, which is common in an immunocompromised host such as a patient with HIV or one undergoing chemotherapy. GI symptoms are the result of ulcers in the esophagus, stomach, small intestine, or colon, which may cause rectal bleeding, bloody diarrhea, or perforation. Ganciclovir is used in the alleviation of symptoms.

The PPD test (choice D) is used to determine whether a patient has had prior exposure to Mycobacterium tuberculosis. Biopsy may show caseating granulomas. In a condition such as this, the patient would also complain of bloody diarrhea and weight loss.

If diagnostic studies are negative and diarrhea persists, patients should undergo endoscopy. Biopsy (choice E) of the duodenum or small bowel may show histologic evidence of cryptosporidial, microsporidial, mycobacterial, or cytomegalovirus (CMV) infection.

How well did you know this?
1
Not at all
2
3
4
5
Perfectly
20
Q

A previously healthy 23-year-old man comes to the physician because of a febrile illness that developed over a 2-day period. He has had temperatures to 39.4 C (102.9 F), with rigors, cough productive of mucopurulent sputum, and right chest pain. At this time, his temperature is 38.7 C (101.7 F), blood pressure is 132/80 mm Hg, pulse is 110/min, and respirations are 22/min. There is no cyanosis. Diminished tactile fremitus, dullness on percussion, and bronchial breathing are present in the right lower lung. A chest x-ray film shows consolidation of the right lower lobe. Microscopic examination of the sputum reveals gram-positive diplococci. The patient denies previous allergic drug reactions. Which of the following is the most appropriate pharmacotherapy?

(A) Cefazolin
(B) Erythromycin
(C) Penicillin
(D) Tetracyclines
(E) Trimethoprim-sulfamethoxazole
(F) Vancomycin

A

Respuesta: C

The correct answer is C. The symptomatology, x-ray evidence of lobar consolidation, and the finding of grampositive diplococci in the sputum all support a diagnosis of acute pneumonia due to pneumococcus. Microscopic examination of gram-stained sputum is more sensitive than culture in identifying pneumococcus. Penicillin is the agent of choice, administered orally (penicillin V) on an outpatient basis in uncomplicated pneumonia, or parenterally (IV penicillin G) for seriously ill patients.

Patients with a history of mild allergic reactions to penicillin, but without anaphylaxis or other serious reactions, should be treated with cefazolin (choice A).

Erythromycin (choice B) is a safe alternative to penicillin for pneumococcal pneumonia and covers the other common bacterial pathogens of community-acquired pneumonia, including (besides pneumococcus) Mycoplasma pneumoniae, Chlamydia pneumoniae, and Legionella.

Tetracyclines (choice D), such as doxycycline, are the preferred drugs against Chlamydia pneumoniae and a good alternative to erythromycin for infections due to Mycoplasma pneumoniae and Moraxella catarrhalis.

The combination of trimethoprim and sulfamethoxazole (choice E) can be used as a second-line treatment in penicillin-allergic patients. It can also be used in case of pneumonia caused by highly penicillin-resistant strains of pneumococcus.

Vancomycin (choice F) is used against strains of pneumococcus highly resistant to penicillin or in case of severe allergic reaction to previous penicillin administration.

How well did you know this?
1
Not at all
2
3
4
5
Perfectly
21
Q

An otherwise healthy 22-year-old woman presents to her physician because of daily headaches over the past 2 weeks. The headaches have a vise-like character, seem to be more intense in the back of the head, and are often precipitated by emotional stress. Physical examination fails to disclose focal neurologic or visual deficits. Which of the following is the most appropriate initial step in patient care?

(A) Antidepressant drugs
(B) Calcium-channel antagonists
(C) Ergotamine-containing preparations
(D) Nonsteroidal anti-inflammatory drugs (NSAIDs)
(E) Sumatriptan

A

Respuesta: D

The correct answer is D. The clinical features of these headaches, with their characteristic vise-like quality, immediately suggest tension headaches, which are often associated with tightness of the neck muscles. Anxiety, fatigue, and noise may act as precipitating triggers. Exploration of underlying causes of anxiety is often useful, but a trial with aspirin or other NSAIDs may be sufficient in most cases. Tension headaches may show overlapping features with migraine.

Antidepressant drugs (choice A) are used to treat depression and its manifestations, including depression headaches. A full neuropsychiatric evaluation is necessary to begin such treatment.

Calcium-channel antagonists (choice B) are effective in decreasing the frequency of attacks of migraine, although they do not influence the intensity and duration of pain.

Ergotamine-containing preparations (choice C) and sumatriptan (choice E) represent the treatments of choice for acute migraine.

22
Q

A 42-year-old woman presents to her physician because of recent urinary tract infections (UTIs). She has been on an unknown oral antibiotic chronically. She has a temperature of 37.2 C (99 F), and costovertebral angle tenderness is noted on the left side. A plain film of the abdomen reveals a radiopaque density filling the left renal pelvis and calyces. Which of the following is the most likely pathogen?

(A) Bacteroides fragilis
(B) Clostridium difficile
(C) Escherichia coli
(D) Proteus mirabilis
(E) Streptococcus bovis

A

Respuesta: D

The correct answer is D. The patient is experiencing recurrent UTIs associated with the presence of kidney stones (the radiopaque density in the renal pelvis and calyces). Urease-producing organisms, such as Proteus mirabilis, create a high urinary pH, contributing to the development of struvite kidney stones. The stone may cause obstruction and urinary stress, leading to infection. These stones are relatively soft and are usually amenable to percutaneous nephrostomy. Acetohydroxamic acid is an effective urease inhibitor. Pseudomonas and Providencia are less common urease-producing organisms that may cause struvite calculi.

Bacteroides fragilis (choice A) is associated with peritonitis in patients with an intra-abdominal abscess.

Clostridium difficile (choice B) is associated with pseudo-
membranous colitis.

Escherichia coli (choice C) is the most common cause of UTI.

Streptococcus bovis (choice E) is a nonenterococcal type of group D Streptococcus.

23
Q

A 35-year-old woman has developed marked thickening of the skin of her hands, particularly her fingers. This thickening is accompanied by hyperpigmentation and is so marked as to limit the range of motion of her fingers. If this patient goes on to develop gastrointestinal problems, which of the following is most likely?

(A) Carcinoid tumor
(B) Duodenal peptic ulcer
(C) Esophageal dysfunction
(D) Pneumatosis cystoides intestinalis
(E) Sacculations of the colon
(F) Small bowel adhesions

A

Respuesta: C

The correct answer is C. The changes seen are those of scleroderma; if other organs become involved, the term systemic sclerosis is appropriate. This disease is characterized by diffuse fibrosis, degenerative changes, and vascular abnormalities. The most common significant internal involvement in these patients is esophageal dysfunction (which may predispose for reflux disease with risk of Barrett esophagus and cancer of the esophagus), which occurs as a result of replacement of the muscle of the esophagus by densely fibrotic, scar-like tissue. Other gastrointestinal complications include pneumatosis cystoides intestinalis (see below), sacculations of the colon and ileum (see below), biliary cirrhosis, and malabsorption secondary to bacterial overgrowth in the poorly functional small bowel.

Carcinoid tumor (choice A) does not have an increased incidence in systemic sclerosis.

Duodenal peptic ulcer (choice B) does not have an increased incidence in systemic sclerosis, although esophageal peptic ulcer, secondary to reflux problems, does.

Pneumatosis cystoides intestinalis (choice D) is an uncommon intestinal complication of systemic sclerosis in which degeneration of the muscularis mucosa allows the entry of air into the intestinal wall.

Sacculations of the colon (choice E) and ileum are broad outpouchings (very fat diverticula) that can sometimes complicate systemic sclerosis as a result of smooth muscle atrophy.

Small bowel adhesions (choice F) are not a feature of systemic sclerosis. They are commonly seen following abdominal surgery (e.g., for appendicitis) and in inflammatory conditions, such as Crohn disease. Small bowel adhesions typically present with symptoms of small bowel obstruction.

24
Q

A 23-year-old man presents with a 3-month history of cough with blood-tinged sputum, shortness of breath, and gross hematuria. His temperature is 37.5 C (99.5 F), blood pressure is 158/94 mm Hg, pulse is 87/min, and respirations are 22/min. Examination reveals bilateral crackles at the lung base and mild edema of the palpebrae and feet. A chest x-ray film shows scattered pulmonary infiltrates in a distribution different from that present on a film taken 2 months ago. Examination of the sputum shows hemosiderin-laden macrophages but no microorganisms. Laboratory investigations show modest iron-deficiency anemia and no evidence of ANCA- type antibodies. Urinalysis shows gross hematuria and modest proteinuria. A renal biopsy demonstrates the presence of glomerulonephritis with linear deposition of IgG and complement components along the glomerular basement membrane. Which of the following is the most likely diagnosis?

(A) Churg-Strauss syndrome
(B) Goodpasture syndrome
(C) Idiopathic pulmonary hemosiderosis
(D) Postinfectious glomerulonephritis
(E) Wegener granulomatosis

A

Respuesta: B

The correct answer is B. Goodpasture syndrome is an autoimmune disease mediated by autoantibodies against a domain of type IV collagen in the basement membranes of both glomerular and alveolar capillaries. Consequently, the lungs develop hemorrhagic interstitial pneumonia manifesting with hemoptysis, whereas the kidneys develop necrotizing glomerulonephritis leading to nephritic syndrome (responsible for hematuria, pitting edema, and hypertension in this case). Linear deposition of IgG and complement along the basement membrane of the alveolar and glomerular capillaries is the pathognomonic feature. The latter alone is sufficient to support a diagnosis of Goodpasture syndrome. Corticosteroids and immunosuppressants are necessary to treat this serious condition.

Churg-Strauss syndrome (choice A) must be considered in the differential diagnosis. This condition is associated with blood and tissue eosinophilia and, frequently, with circulating ANCA, i.e., antineutrophil cytoplasmic antibodies (specifically p-ANCA).

Idiopathic pulmonary hemosiderosis (choice C) may appear similar to Goodpasture syndrome in its pulmonary manifestations- hemoptysis and pulmonary infiltrates-but this condition does not involve the kidneys nor is it associated with linear IgG deposition along basement membranes.

Postinfectious glomerulonephritis (choice D) most commonly follows a streptococcal infection and manifests with nephritic syndrome. Pulmonary manifestations are not present. Immunofluorescence of kidney biopsies reveals granular deposition of IgG and complement in the mesangium and glomerular basement membrane.

Wegener granulomatosis (choice E) enters the differential diagnosis of any condition manifesting with concomitant involvement of lungs and kidneys. It is characterized by a necrotizing granulomatous vasculitis and frequent presence of circulating c-ANCA.

25
A 67-year-old man comes to the physician because of insomnia, irritability, and palpitations for 3 months. He is currently taking amiodarone for cardiac arrhythmias, fluoxetine for depression, and enalapril for hypertension. His blood pressure is 130/70 mm Hg, and his pulse is 90/min and regular. Which of the following is the most appropriate next step? (A) Measurements of thyroxine and TSH (B) Administration of propranolol (C) Referral for psychiatric consultation (D) Substitution of antidepressant drug (E) Substitution of antihypertensive drug
Respuesta: A The **correct answer is A**. Insomnia, irritability, and palpitations are nonspecific symptoms that may be caused by a variety of diseases and drugs, but they are frequent manifestations of hyperthyroidism. Furthermore, the fact that the patient takes amiodarone should prompt investigations for hyperthyroidism. Amiodarone causes symptomatic hyperthyroidism in a small percentage of patients (2 to 3%) and asymptomatic elevation of T3 and T4 with much greater frequency. Thus, thyroid hormone measurements should be combined with measurement of TSH, which is suppressed in the presence of significant thyroid hyperfunction. Administration of propranolol **(choice B)** is effective in relieving symptoms of hyperthyroidism due to abnormal sympathetic activation, namely tachycardia, excessive sweating, anxiety, and tremor. It should be used for temporary relief until hyperthyroidism has resolved, but is not adequate treatment in this case. Referral for psychiatric consultation **(choice C)** implies that the symptoms are due to an underlying psychiatric etiology, which is a plausible explanation in a patient with history of depression. However, hyperthyroidism should be ruled out first. Substitution of a different antidepressant drug **(choice D)** would be justified if the symptoms were due to fluoxetine administration. Treatment with fluoxetine, as well as other serotonin-selective reuptake inhibitors (SSRI), may cause insomnia and nervousness. Again, amiodarone-related hyperthyroidism should be ruled out before attributing the symptoms to the SSRI side effects. Substitution of a different antihypertensive drug **(choice E)** would not be justified in this case. Enalapril, as any other angiotensin- converting enzyme (ACE) inhibitors, is a remarkably safe drug with few and rare adverse effects. Hypotension is one of these, but the patient in this example has a blood pressure within a fairly normal range.
26
An unconscious 35-year-old man is brought to the emergency department by his wife. She explains that the patient takes phenytoin for chronic epilepsy. An hour ago, the patient had a seizure but did not regain consciousness. Physical examination reveals that his temperature is 38.5 C (101.3 F), blood pressure is 92/40 mm Hg, pulse is 110/min, and respirations are 20/min. During the examination, the physician observes the sudden onset of tonic-clonic convulsions. Which of the following is the most common precipitating cause of this emergency? (A) Alcohol withdrawal (B) Drug noncompliance (C) Head trauma (D) Hypoxia (E) Intracranial infection
Respuesta: B The **correct answer is B**. Status epilepticus is a life-threatening emergency that should be treated promptly. It is diagnosed when a generalized convulsive seizure lasts longer than 10 minutes or when a seizure episode is followed by another episode without recovery of consciousness. There are two types of status epilepticus: convulsive and nonconvulsive. The convulsive type is the most dangerous. It can lead to metabolic and cardiovascular disturbances, including hypoxemia, hypoglycemia, hypotension, and hyperthermia, that may cause death or permanent brain damage. About 50% of patients presenting with status epilepticus do not have history of epilepsy. The most frequent precipitating factor in adults with a diagnosis of epilepsy is drug noncompliance. Alcohol withdrawal **(choice A)**, head trauma **(choice C)**, hypoxia **(choice D)**, intracranial infection **(choice E)**, intracranial tumor **(choice F)**, and metabolic alterations **(choice G)** are other precipitating factors for status epilepticus. Of these, infection is the most common in childhood.
27
A 26-year-old librarian presents with chronic daytime somnolence, which has frequently caused him to fall asleep at work. He does not smoke but drinks 1 to 2 glasses of wine daily. He says he frequently awakens at night but denies any visual or auditory hallucinations on falling asleep. His height is 186 cm (73 in), and his weight is 60% greater than expected. Chest examination reveals no specific findings other than distant breath sounds. Arterial blood gas analysis during normal ventilation shows: - Pao2: 82 mm Hg - Paco2: 55 mm Hg After the patient voluntarily hyperventilates for 1 minute, blood gas analysis returns within normal limits. Which of the following will have the greatest benefit on this patient’s symptoms? (A) Benzodiazepines at bedtime (B) Daily acetazolamide (C) Morning administration of dextroamphetamine (D) Supplemental oxygen at night (E) Weight loss
Respuesta: E The **correct answer is E**. The clinical picture and results of blood gas analysis before and after hyperventilation are characteristic of obesity-hypoventilation syndrome (also known as pickwickian syndrome, after a character in Charles Dickens’ The Pickwick Papers). Hypoventilation results from a combination of reduced drive on respiratory centers and physical impediment on respiration imposed by obesity. Improvement of hypoxemia and hypercapnia following voluntary hyperventilation differentiates this condition from chronic obstructive pulmonary disease. Most patients with pickwickian syndrome also have obstructive sleep apnea and consequent daytime sleepiness. Weight loss is the single most effective therapeutic intervention. Benzodiazepines at bedtime **(choice A)** are contraindicated, as are any other hypnotic agents. Alcohol should also be avoided. Treatment with daily acetazolamide **(choice B)** has been tried in obstructive sleep apnea but with disappointing results. Morning administration of dextroamphetamine **(choice C)** is used for the treatment of narcolepsy. This disease is hereditary and manifests with sudden sleep attacks, cataplexy (abrupt loss of muscle tone), and hypnagogic hallucinations. None of these symptoms are present in this case. Supplemental oxygen at night **(choice D)** has been found to have some benefit in reducing the severity of nocturnal episodes of hypoxemia in obstructive sleep apnea, but it may also increase the duration of apneic episodes.
28
A 35-year-old man with a history of chronic heroin abuse comes to the physician because of progressive swelling of his feet and hands. His blood pressure is 155/90 mm Hg. Laboratory studies show: - Creatinine 1.6 mg/dL - BUN 20 mg/dL - Urinalysis Protein 4+ - Erythrocytes 10/hpf The amount of protein measured in a 24-hour urine collection is 4.5 g. Which of the following is the most likely diagnosis? (A) Acute proliferative glomerulonephritis (B) Crescentic glomerulonephritis (C) Focal segmental glomerulosclerosis (D) Minimal change disease (E) Nodular glomerulosclerosis (Kimmelstiel-Wilson disease)
Respuesta: C The **correct answer is C**. The clinical presentation is consistent with nephrotic syndrome, since proteinuria is within the nephrotic range, i.e., >3 g/day. The history of heroin abuse makes focal segmental glomerulosclerosis (FSG) the most likely diagnosis. FSG may occur in an idiopathic form or in association with three conditions: morbid obesity, HIV infection, and heroin abuse. A renal biopsy will reveal sclerosis occurring in some, but not all, glomeruli (focal), with each glomerulus showing partial involvement (segmental). Electron microscopy shows detachment of epithelial podocytes from the glomerular basement membrane, an alteration also seen in minimal change disease. Note in this clinical case, the coexistence of nephrotic features (marked proteinuria and edema) with nephritic signs (hypertension and microhematuria), which is often present in FSG. Acute proliferative glomerulonephritis **(choice A)** is characterized by proliferation of endothelial and mesangial cells with influx of leukocytes. The glomeruli are hypercellular, and immune deposits are epimembranous in location. This pattern is associated with nephritic syndrome, i.e., proteinuria < 3 g/day, hematuria, hypertension, and pedal and periorbital edema. The prototype of this glomerular disease is postinfectious glomerulonephritis. Crescentic glomerulonephritis **(choice B)** owes its designation to the crescent-shaped masses of cells (epithelial and inflammatory) that accumulate within the urinary space of Bowman capsule, obliterating the glomerular tuft. This results in a rapidly progressive renal failure, requiring aggressive immunosuppressive therapy. Minimal change disease **(choice D)** is mostly a disease of childhood. It manifests with full-blown nephrotic syndrome. On light microscopy, the glomeruli appear normal, only to reveal alterations in epithelial podocytes on electron microscopy. Nodular glomerulosclerosis **(Kimmelstiel-Wilson disease; choice E)** is pathognomonic of diabetic nephropathy. Round PAS-positive (i.e., glycoprotein-rich) globules are seen within the glomeruli. This feature, along with diffuse mesangial sclerosis due to accumulation of altered glycoproteins of plasma origin, constitutes the pathologic substrate of diabetes-related renal dysfunction.
29
A 61-year-old man presents for an elective surgical incision and drainage procedure. The patient has an 8-year history of hepatitis C infection with well-documented cirrhosis and portal hypertension. He has a large hematoma on his thigh that is suspected to have necrotic tissue underlying it and therefore requires debridement. On preoperative screening, his prothrombin time is noted to be 17.4 seconds. Transfusion of which of the following is the most appropriate next step in management of this patient prior to his procedure? (A) Cryoprecipitate (B) Fresh frozen plasma (C) Packed red blood cells (D) Platelets (E) Whole blood
Respuesta: B The **correct answer is B**. A basic understanding of blood product and blood component replacement is crucial. The use of such products is extremely common, and there is misuse. Patients with liver disease have a deficiency of one or more clotting factors produced by the liver. A blood product that specifically raises such factors is indicated for treatment. Fresh frozen plasma (FFP) generally increases plasma anticoagulation factors by 30%. Like all blood products, it is type specific. There is a correlation for prothrombin times greater than 15 and the risk of bleeding with invasive procedures such as paracentesis. For this reason, FFP is usually indicated in such patients prior to undergoing their procedure. Cryoprecipitate **(choice A)** is prepared from FFP and contains concentrated factor VIII, factor XIII, fibrinogen, and von Willebrand factor. Indications for use are hypofibrinogenemia (DIC), von Willebrand disease, and hemophilia A. Packed red blood cells **(choice C)** are prepared from all the red cell mass in a pint of donated blood. It has no plasma or buffy coat and therefore no proteins (coagulation factors) or platelets. It is used to restore red cell mass. Platelets **(choice D)** are a blood component therapy used to restore platelet count. One unit of platelets increases the platelet count by 5000-10,000 cells/mm3 , assuming no ongoing destruction or sequestration. Platelets are usually transfused as a “six-pack.” Each unit is the product of one unit donated whole blood; thus, a “six- pack” represents pooled platelets from multiple donors. Transfusion of whole blood **(choice E)** is not a current practice. Whole blood is the content of 1 pint of donated blood. It is unfiltered and contains plasma, platelets, white cells, and red cells. This product is usually processed so that each of these components is removed (except white cells) and used for transfusions in specific clinical situations.
30
A 55-year-old woman with long-standing diabetes mellitus and a 2-year history of progressive renal failure comes to medical attention because of chest pain for 12 hours. The pain is substernal and continuous, with radiation to the neck. She is on a strict dietary regimen with protein, fluid, and salt restriction. Her temperature is 37.2 C (99 F), blood pressure is 150/85 mm Hg, pulse is 82/min and regular, and respirations are 16/min. There is no jugular vein distention or pitting edema. Auscultation reveals a rubbing sound in the precordial region and slightly distant but normal heart sounds. Lungs are clear to auscultation. The patient is admitted, and laboratory studies show: - Hematocrit: 33% - Hemoglobin: 11.2 g/dL - Leukocyte count: 12,500/mm3 - BUN: 102 mg/dL - Serum glucose: 128 mg/dL - Na: 142 mEq/L - K: 5.3 mEq/L - Cl: 103 mEq/L *Arterial blood - pH 7.38 - Po2 92 mm Hg - Pco2 39 mm Hg A chest x-ray film shows a normal cardiac outline, and an ECG shows nonspecific ST changes. Echocardiogram reveals mild fluid collection within the pericardial sac. Which of the following is the most appropriate next step in management? (A) Water and salt intake reduction (B) Antibiotic treatment (C) Antihypertensive treatment (D) Anti-inflammatory treatment (E) Erythropoietin administration (F) Hemodialysis (G) Pericardiocentesis (H) Pericardial biopsy (I) Partial pericardiectomy
Respuesta: F The **correct answer is F**. A friction rub on auscultation indicates that the patient’s chest pain is due to acute fibrinous pericarditis. ECG changes in this condition are often nonspecific, but echocardiography is a sensitive diagnostic tool. Renal failure is one of the most common causes of acute pericarditis (uremic pericarditis), which usually occurs when BUN exceeds 100 mg/dL (often earlier in diabetic patients). Fever is usually absent in uremic pericarditis. Institution of hemodialysis (or more aggressive hemodialysis) promptly leads to resolution of pericarditis. Indeed, the onset of acute pericarditis is an absolute indication to start hemodialysis treatment. Further reduction of water and salt intake **(choice A)** would not be sufficient to treat uremic pericarditis and might be counterproductive in this specific case. Antibiotic treatment **(choice B)** is useful in cases of infective (purulent) pericarditis, but uremic pericarditis is the result of circulating toxins, not infection. Antihypertensive treatment **(choice C)** is often necessary in renal failure but has no effect on uremic pericarditis. In this case, the blood pressure is within “borderline” values. Anti-inflammatory treatment **(choice D)** is helpful in reducing symptoms but does not affect the natural course of the process. Indomethacin or corticosteroids may be used. Erythropoietin administration **(choice E)** is used to treat anemia of renal failure. It is usually started when hematocrit falls below 30 to 35%. Pericardiocentesis **(choice G)** and partial pericardiectomy (“pericardial window"; choice I) are used to treat pericardial tamponade, which is an accumulation of large amounts of pericardial fluid (inflammatory or hemorrhagic) that impairs diastolic filling and causes acute heart failure. In this case, the amount of exudate within the pericardium and the hemodynamic conditions do not warrant such therapy. Pericardial biopsy **(choice H)** is advisable when the etiology is not clear, which is not the case in the presence of an obvious clinical picture of renal failure.
31
A 28-year-old man comes to the emergency department complaining of abdominal pain. He has no significant past medical history, has had no recent illnesses, and denies any alcohol or drug abuse. He reports that 3 days ago, he developed acute pain in his right upper quadrant. The pain was nonradiating and was associated with nausea and two episodes of nonbloody, nonbilious emesis. He also reports that 2 days ago, he began to turn “yellow.” On examination, he is afebrile and has scleral icterus with mild jaundice of his skin. His right upper quadrant is tender, with no palpable gallbladder and no Murphy’s sign. Determination of which of the following is the most appropriate next step in diagnosis? (A) Serum hepatitis A IgG titer (B) Serum hepatitis A IgM titer (C) Serum hepatitis B surface antibody titer (D) Serum hepatitis C antibody (E) Serum hepatitis C RNA level
Respuesta: B The **correct answer is B**. This patient likely has acute hepatitis A infection. The prodrome of this infection is very similar to this patient’s presentation, and within 10-14 days after infection, many patients will manifest varying degrees of abdominal pain as well as jaundice. The disease is self-limiting, usually transmitted by contaminated shellfish or oral-anal contact with an infected person or their feces, and does not predispose patients to the same longterm risks as infection with the other hepatitis viruses. In the acute setting, serum IgM antibody may be positive. A positive serum hepatitis A IgG titer **(choice A)** would be seen months after the acute infection has passed and is a marker for previous infection. A positive serum hepatitis B surface antibody titer **(choice C)** is a marker for previous hepatitis B exposure. A positive serum hepatitis C antibody **(choice D)** is a marker for hepatitis C infection. Although both B and C varieties can cause acute viral illnesses similar to the one in this patient, the epidemiology of their transmission is quite different from this patient’s risk factors. Both agents are transmitted via blood-to-blood contact, and current epidemiology indicates this is primarily by IV drug abuse exposure to infected blood (e.g., health care workers and needle sticks). Unlike hepatitis A, which has no long-term sequelae and is self-limited, both hepatitis B and C infections are associated with significant long-term morbidity and mortality. Hepatitis C is the most common cause of nonalcoholic cirrhosis and liver failure. A positive serum hepatitis C RNA level **(choice E)** is a test ordered after initial exposure to hepatitis C and is used to follow the activity of disease over time. Unless hepatitis C infection was suspected in this patient, this test is not appropriate.
32
A 40-year-old woman comes to the physician because of a 6-month history of increasing respiratory difficulty that occurs during mild exercise, such as walking uphill. She is 165 cm (65 in) tall and weighs 58 kg (129 lb), but says that she has lost 4 kg (9 lb) over the past 3 months. Her blood pressure is 120/75 mm Hg, pulse is 85/min, and respirations are is 16/min. Chest examination reveals crackles at both lung bases and a diastolic murmur near the cardiac apex. The characteristics of the murmur change with the patient’s position. Echocardiography reveals a solid mass that partially fills the left atrium and results in obstruction of the mitral flow. Which of the following is the most likely diagnosis? (A) Fibroelastoma (B) Metastasis (C) Mural thrombus (D) Myxoma (E) Sarcoma
Respuesta: D The **correct answer is D**. Myxoma is the most common primary cardiac tumor. It affects women more frequently than men and grows in the left atrium in 80% of cases. It may present with a systemic illness mimicking infective endocarditis, give rise to systemic embolism, or present with signs and symptoms of mitral valve obstruction (as in this case). The change in murmur when the patient changes position is suggestive of an atrial myxoma. The tumor sometimes produces a diastolic sound consequent to the tumor motion (tumor plop). Echocardiography is diagnostic. Fibroelastoma **(choice A)** results from organization of a mural thrombus and is usually clinically silent. It represents an incidental postmortem finding. Metastasis to the heart **(choice B)** is a relatively rare event that affects the pericardial sac or ventricular wall. Melanoma is the most common malignancy metastasizing to the heart. In addition, malignant neoplasms from the lungs, breast, pleura, and mediastinal organs may involve the heart. A mural thrombus **(choice C)** is a frequent complication of abnormalities in wall motion resulting, for example, from myocardial infarction or atrial fibrillation. Mural thrombosis, however, does not appear as a mass filling the left atrium and does not produce mitral valve obstruction. Its most significant risk is embolization. Sarcoma of the heart **(choice E)** is an exceptionally rare event that would not manifest with an intra-atrial mass.
33
A 45-year-old woman with rheumatoid arthritis develops pain, erythema, and swelling of the cartilaginous portion of both of her external ears. This is accompanied by pain localized to the costochondral joints. Which of the following is the most likely diagnosis? (A) Ankylosing spondylitis (B) Behcet syndrome (C) Gout (D) Reiter syndrome (E) Relapsing polychondritis
Respuesta: E The **correct answer is E**. The patient has relapsing polychondritis, which is an autoimmune condition that occurs as an isolated process or together with other autoimmune diseases, including rheumatoid arthritis, systemic vasculitis, and systemic lupus erythematosus. Presentations can include bilateral swelling of the external ears, nasal involvement, arthralgias to symmetric arthritis (with a predilection for the costochondral joints), and involvement of the larynx, trachea, and bronchi. The condition can also involve the cardiovascular system, kidney, and skin. In severe cases, the cartilage destruction may be disfiguring (floppy ears, saddle nose). Ankylosing spondylitis **(choice A)** causes arthritis of the lower back. Behcet syndrome **(choice B)** is characterized by oral ulcers, genital ulcers, and arthritis. Gout **(choice C)** can cause both arthritis and ear involvement with tophi, but has no particular predilection for the costochondral joints. Reiter syndrome **(choice D)** produces urethritis and arthritis, sometimes related to Chlamydia infection.
34
An otherwise healthy 40-year-old woman comes to the physician because she discovered a painless nodule in her neck. On physical examination, palpation reveals a firm 1-cm nodule in the left cervical region, which moves upward as the patient swallows. Thyroid function tests are normal. Fine needle aspiration is positive for papillary carcinoma. Which of the following is a recognized risk factor for this form of cancer? (A) Amiodarone treatment (B) Dietary iodine supplementation (C) Family history of multiple endocrine neoplasia (MEN) (D) Graves disease (E) Hashimoto thyroiditis (F) Radiation to the neck (G) Subacute thyroiditis (H) Thyroglossal duct anomalies
Respuesta: F The **correct answer is F**. Papillary carcinoma is the most common variant of thyroid carcinomas, and the one associated with the best prognosis. Previous radiation to the neck, whether iatrogenic (for Hodgkin lymphoma, acne, tonsillitis, etc.) or resulting from accidental exposure, is a risk factor for both papillary carcinoma and follicular carcinoma (the latter being the second most common type). The frequency of thyroid cancer is also increased in areas of high goiter incidence. Amiodarone treatment **(choice A)** is associated with a 2 to 3% incidence of hyperthyroidism in patients treated with this drug, but not with thyroid cancer. Dietary iodine supplementation **(choice B)** in salt and water has become standard practice in areas of low iodine concentration as a public health measure to prevent goiter. This has virtually eliminated goiter due to environmental iodine deficiency. A family history of multiple endocrine neoplasia (MEN) **(choice C)** is not a risk factor. None of the three types of MEN is associated with papillary or follicular carcinoma of the thyroid. MEN types IIa and IIb predispose to medullary carcinoma of the thyroid, which originates from C cells. Clinical evidence linking either Graves disease **(choice D)** or Hashimoto thyroiditis **(choice E)** to thyroid cancer is still controversial, but it is safe to assume that if there is any increased risk for patients with either disorder, it is very small. Subacute thyroiditis **(choice G)**, also known as de Quervain thyroiditis, manifests as painful enlargement of the thyroid, accompanied by dysphagia. It does not confer any increased risk for thyroid cancer. Thyroglossal duct anomalies **(choice H)** include thyroglossal cysts and fistulas. These embryologically related conditions do not increase the risk of thyroid cancer. Exceptionally, squamous cell carcinoma has been reported arising in one of these anomalies.
35
A 40-year-old man is admitted for chemotherapy for treatment of acute myelogenous leukemia. A central line through his subclavian vein is inserted to facilitate infusion of chemotherapeutic agents. Ten days after this procedure, he develops a temperature of 39.4 C (103.0 F). Physical examination is remarkable for tachycardia and tenderness around the central line insertion site. Blood cultures and a chest x-ray film are negative. Which of the following is the most appropriate next step in management? (A) Repeat blood cultures and wait for culture results to guide therapy (B) Administer amphotericin (C) Administer vancomycin (D) Remove the central line (E) Remove the central line and insert a new one over a guide wire
Respuesta: D The **correct answer is D**. Unfortunately, nosocomial infections are very common in hospitals. Given this patient’s physical examination and history, it is most likely that the central line is the site of the current infection. Anything short of removal of the line will not remove the source of the infection, and it will be almost impossible to treat the infection. Waiting until the blood cultures grow the offending organism **(choice A)** would be risky since the patient’s infection would remain untreated and get out of control. If the infecting pathogen is not known, broad coverage is preferred; therapy can be narrowed once the pathogen, and its sensitivity to antibiotics, is known. Administering an antifungal agent, such as amphotericin **(choice B)**, would be an option if this patient were septic from an infection such as Candida or Aspergillus. This is likely in this patient with leukemia; his immunity is probably suppressed, and he is prone to fungal infections. However, treatment with amphotericin would not address the underlying problem related to the indwelling central line. Administering vancomycin **(choice C)** would be helpful if this patient were infected with Staphylococcus, which is likely in the setting of a line infection. In fact, vancomycin is preferred in this setting; however, it will not be successful until the contaminated line is removed. Replacement over the guide wire **(choice E)** will simply re- introduce the line infection.
36
A recently widowed 35-year-old woman who has an anxiety disorder, for which she has been treated with alprazolam and imipramine, is brought to the emergency department for a multiple drug overdose. Her daughter states that she may have taken 30-40 tablets of 50-mg imipramine and 35-40 tablets of 1-mg alprazolam. On route to the hospital, she became apneic and was intubated. Her blood pressure is 130/84 mm Hg, and her pulse is 120/min. The patient is unresponsive to painful stimuli, and her pupils react very sluggishly, but there are no other neurologic findings. The ECG shows a normal sinus rhythm with a widened QRS complex. Which of the following is the most appropriate intervention? (A) DC cardioversion (B) Flumazenil (C) Ipecac (D) Lidocaine bolus (E) Maintenance of serum pH at 7.5
Respuesta: E The **correct answer is E**. Any unknown overdose with QT prolongation on ECG should raise the suspicion of tricyclic antidepressant (TCA) overdose. We know that the patient ingested both TCAs and benzodiazepines in overdose amounts. The ECG changes (QRS widening) are signs of TCA intoxication. The treatment is to maintain an alkalemic state, by hyperventilation if the patient is intubated or with IV bicarbonate. Gastric aspiration and lavage should be performed to eliminate unabsorbed drug if more than 750 mg of TCA has been taken. Seizures may occur in TCA overdose, but this patient also took an overdose of a benzodiazepine, which is likely suppressing the seizures. DC cardioversion **(choice A)** would not be indicated, since this patient is in sinus rhythm. Flumazenil **(choice B)** would reverse the effects of the benzodiazepine and might precipitate life-threatening seizures. Ipecac **(choice C)** would be contraindicated in a comatose patient. Gastric aspiration and lavage would be more suitable. Lidocaine **(choice D)** is indicated for ventricular dysrhythmias, but this patient was in sinus rhythm.
37
A 38-year-old woman visits the clinic because of oral tenderness and soreness in her jaw for the past 2 months. She admits that swallowing food has become more painful. She notices that her mouth is chronically dry and that drinking lots of fluids seems to reduce the pain. She has a normal appetite but reports that she has lost 2 kg (5 lb) and is eating less because of the pain upon swallowing most foods. Physical examination reveals parched lips, dry oral mucous membranes, and bilaterally enlarged parotid glands with a firm, smooth texture. Needle biopsy of the salivary gland tissue reveals a dense, lymphocytic infiltrate with loss of many glands. Residual glands have prominent intraductal cellular proliferation. Which of the following is the most likely diagnosis in this patient? (A) Mucoepidermoid carcinoma (B) Pleomorphic adenoma (C) Sarcoidosis (D) Sjögren syndrome (E) Squamous cell carcinoma (F) Warthin tumor
Respuesta: D The **correct answer is D**. This is Sjögren syndrome, which is an autoimmune condition that can damage the salivary glands and/or the tear glands. The presence of dry eyes or dry mouth is common, but be aware that about one third of patients have significant parotid gland enlargement (one or both sides) and may present with parotid masses and dry mouth with pain upon swallowing. In cases where a parotid mass is not present, biopsy of the lip may show characteristic changes in minor salivary glands, without the risk of damage to the facial nerve. Patients with Sjögren syndrome may have other autoimmune diseases as well, notably rheumatoid arthritis, scleroderma, and systemic lupus erythematosus. Mucoepidermoid carcinoma **(choice A)** is characterized by nests of tumor cells without a dense lymphocytic infiltration. Pleomorphic adenoma **(choice B)** is characterized by a variety of histologic patterns with nests, cords, or sheets of benign tumor cells. Sarcoidosis **(choice C)** can destroy salivary glands but is characterized by prominent granulomas. Squamous cell carcinoma **(choice E)** typically presents as an asymptomatic, white oral patch in a person who smokes, chews tobacco, or drinks alcohol. There is no associated parotid gland enlargement or salivary gland damage. Warthin tumor **(choice F)** is a tumor composed of lymphoid tissue resembling tonsils that is covered by a distinctive, two-cell deep epithelium.
38
A 52-year-old man with a 30-pack-year history of cigarette smoking presents to a physician after moving to a new city. He was told that he had “high cholesterol” about 2 years ago, and he has a history of mild hypertension for which he has never been treated. He had a myocardial infarction (MI) 6 months ago. His post-MI course has been uncomplicated, his exercise stress test was satisfactory, and he has experienced no subsequent chest pain. His medications include one aspirin tablet every other day. Physical examination is normal except for a fourth heart sound. Which of the following is the most appropriate next step in management to prevent significant morbidity and mortality? (A) Add a beta blocker (B) Add enalapril (C) Add nifedipine (D) Increase the aspirin to one tablet three times daily (E) Prescribe nitroglycerin for angina
Respuesta: A The **correct answer is A**. Beta blockers have been shown to decrease the incidence of nonfatal reinfarction and recurrent ischemic events. They decrease both infarct size and mortality. Enalapril **(choice B)** is recommended for the first 6 weeks in patients with a large anterior wall infarction to decrease mortality by preventing infarct remodeling and expansion, although angiotensin converting enzyme (ACE) inhibitors may be harmful if hypotension is present. Calcium-channel blockers, especially diltiazem and verapamil, may be beneficial if the ejection fraction is adequate. Nifedipine **(choice C)** has been shown to increase mortality in patients following an MI. The patient is already taking one aspirin every other day. Increasing the dose to three tablets daily **(choice D)** will not help and, in fact, may hurt him. Adding nitroglycerin **(choice E)** would be appropriate if the patient developed angina, but at present he has none.
39
A 17-year-old boy of Jewish descent is taken to the emergency department by his mother because of bloody diarrhea. Over the past 2 weeks, the boy has reported frequent urges to defecate that are accompanied by abdominal cramping. Over the past several days, the stools have become looser, and mucus was present around the feces. One hour ago, he saw fresh blood on his stool. On questioning, the boy notes that similar symptoms have occurred over the past 2 years, except for the blood in his stool. His temperature is 37.5 C (99.5 F), blood pressure is 120/70 mm Hg, pulse is 65/min, and respirations are 16/min. His abdomen is soft, without guarding, and there is localized tenderness in the right lower quadrant. Which of the following is the most likely diagnosis? (A) Appendicitis (B) Colon cancer (C) Diverticulitis (D) Mesenteric lymphadenitis (E) Pseudomembranous colitis (F) Ulcerative colitis
Respuesta: F The **correct answer is F**. Ulcerative colitis has a bimodal distribution of age of onset, with a large peak between ages 15 and 30 and a smaller peak between ages 50 and 70. It occurs in both sexes and all races, although there is a somewhat increased rate in the Jewish population. The underlying etiology of the condition is still poorly defined. The presentation illustrated is typical, and many patients have actually had developing disease for several years before the diagnosis is made. The process usually starts in the rectum, and the stool may be hard unless the involved segment of bowel is extensive. Stool mixed with mucus and fresh blood is typical of an exacerbation. With severe disease, accompanying findings can include malaise, fever, anemia, anorexia, weight loss, and leukocytosis. The diagnosis should be confirmed by sigmoidoscopy. Appendicitis **(choice A)** can occur in teenagers but usually causes left lower quadrant pain and would not produce stool with blood and mucus. Colon cancer **(choice B)** can produce similar symptoms but is extremely rare among teenagers. Diverticulitis **(choice C)** can produce similar symptoms, but it is a disease of middle-aged to elderly individuals and is not associated with rectal bleeding. Mesenteric lymphadenitis **(choice D)** refers to inflammation of the mesenteric lymph nodes. It often presents acutely in children and is clinically difficult to differentiate from acute appendicitis and enterocolitis. Clinical features of an associated enterocolitis or ileitis in a Yersinia infection, such as right lower quadrant pain, fever, diarrhea, nausea, and vomiting, may be present. However, bloody stools are not a feature of mesenteric lymphadenitis. Pseudomembranous colitis **(choice E)** is due to a clostridial infection that is usually seen as a complication of broad-spectrum antibiotic use.
40
A 32-year-old man with AIDS has a CD4+ T-cell count of 40/mm3. He complains of a gradual onset of decreased vision in his right eye over the past few days. He is afebrile and has temporal wasting. There is a thick, cheesy, white exudate on his tongue and oropharynx, and there are deficits in the acuity and the visual fields of his right eye. The remainder of the cranial nerve examination is normal. Infection with which of the following pathogens is the most likely cause of his decreased vision? (A) Candida albicans (B) Cytomegalovirus (CMV) (C) Herpes simplex virus 1 (HSV-1) (D) Mycobacterium avium-intracellulare (MAI) (E) Pneumocystis carinii (F) Toxoplasmosis
Respuesta: B The **correct answer is B**. This patient has a very low CD4+ count, predisposing him to a variety of opportunistic infections. This patient’s visual symptoms are most likely due to cytomegalovirus (CMV) retinitis, the most common cause of HIV-associated retinitis. Other manifestations of CMV infection include constitutional symptoms, gastrointestinal disturbances, bone marrow suppression, adrenalitis, and lower respiratory tract infections. CMV is treated with ganciclovir or foscarnet. Although the thick, cheesy white exudate on the tongue and oropharynx is likely due to Candida albicans **(choice A)**, which is also common in AIDS patients, this pathogen does not cause visual deficits. Herpes simplex virus 1 **(HSV-1; choice C)** causes herpes labialis and keratitis. Herpes keratitis occurs initially as conjunctivitis with vesicular blepharitis. Recurrences are termed “dendritic keratitis” and are characterized by branched corneal lesions that look like the veins of a leaf. A foreign body sensation, along with lacrimation, photophobia, and conjunctival hyperemia, may occur at the onset of the condition. AIDS patients with CD4+ counts < 100/mm3 are particularly susceptible to infection with M. avium-intracellulare **(MAI; choice D)**. For this reason, such patients are routinely administered MAI prophylaxis with rifabutin. The most common manifestations of MAI infection are fever of unknown origin, weight loss, and gastrointestinal disease. Pneumocystis carinii **(choice E)** is a common cause of pneumonia in HIV+ patients and is often the first presenting sign of AIDS. Patients with CD4+ counts < 200/mm3 should receive prophylaxis against P . carinii pneumonia (PCP) with double-strength trimethoprim-sulfamethoxazole (TMP-SMX). In sulfa-allergic patients, dapsone or pentamidine may be used. Toxoplasmosis **(choice F)** is a rare cause of chorio-retinitis in AIDS patients, but the most likely cause of this patient’s ocular changes is still CMV. When Toxoplasma does affect the eyes, it causes decreased vision, eye pain, and necrotizing lesions seen on fundoscopy. Most commonly, Toxoplasma affects the brain, with or without focal lesions. Clinical findings in the CNS include an altered mental state, seizures, weakness, cranial nerve disturbances, sensory abnormalities, cerebellar signs, meningismus, movement disorders, and neuropsychiatric manifestations.
41
A 5-year-old boy is brought to the emergency department 4 hours after sudden onset of fever and chills. He was bitten in his right hand by the family’s dog 24 hours ago. Examination reveals superficial lacerations of the ulnar aspect of the right hand consistent with a history of dog bite. The wound is surrounded by extensive skin erythema and soft tissue swelling. Palpable lymph nodes are found in the right axilla. His temperature is 38.9 C (102F). Which of the following is the most likely pathogen or pathogens? (A) Capnocytophaga canimorsus (B) Eikenella corrodens (C) Mixed aerobic and anaerobic bacteria (D) Pasteurella multocida (E) Staphylococci (F) Streptococci
Respuesta: D The **correct answer is D**. Dog bites are the least likely to become infected compared with cat and human bites. The infection rate for dog bites is only 5%, for cat bites 30 to 50%, and an intermediate figure for human bites. The pathogens accounting for infection are different depending on the biting animal and the time of onset of infection. Early infections (in the first day) due to dog or cat bites are usually secondary to Pasteurella multocida. This agent is sensitive to penicillin or tetracyclines; however, the response is slow, and treatment should be continued for at least 2 weeks. Early infections following human bites are usually due to mixed aerobic and anaerobic bacteria **(choice C)**, which are normal components of the oral flora. Capnocytophaga canimorsus **(choice A)**, Eikenella corrodens **(choice B)**, and especially staphylococci **(choice E)** and streptococci **(choice F)** are responsible for late infections, occurring more than 24 hours after a bite. Capnocytophaga canimorsus is a gram- negative organism of the canine mouth flora, whereas Eikenella corrodens is a saprophyte of the human mouth. The pathogens involved in human bite infections are so variable that therapy should be adjusted once antibiotic sensitivity has been determined on the pathogen(s) isolated from cultured wounds.
42
A 65-year-old woman consults a physician with complaints of severe pain and stiffness of the neck, shoulders, and hips. These symptoms are worst in the morning and after inactivity. The woman has also been experiencing a variety of systemic symptoms, including malaise, lowgrade fever, depression, and some weight loss. On physical examination, there is no evidence of erosive or destructive joint disease, no point tenderness when pressure is applied to small joints, no selective muscle weakness or muscle atrophy, and no rheumatoid nodules. Blood studies demonstrate a mild normochromic normocytic anemia, dramatically elevated erythrocyte sedimentation rate, and elevated c-reactive protein. Thyroid-stimulating hormone (TSH) is 0.75 mIU/mL. Rheumatoid factor is negative. Electromyography and muscle biopsy fail to demonstrate conclusive evidence of muscle disease. This patient’s most likely condition has the strongest association with which of the following? (A) Crohn disease (B) Dermatomyositis (C) Discoid lupus erythematosus (D) Sjögren syndrome (E) Temporal arteritis
Respuesta: E The **correct answer is E**. This patient has polymyalgia rheumatica, which affects older individuals with a 2 to 1 female to male predominance. The disease is characterized by both severe muscle pain with stiffness and usually prominent systemic symptoms, such as malaise, fever, and weight loss. The muscle pain and stiffness tend to involve areas near the trunk and are not accompanied by muscle wasting or evidence of muscle damage on electromyography or biopsy. This condition is associated with temporal arteritis (which can cause blindness). Patients should be warned to inform their physician promptly if they develop severe headaches, visual changes, or significant jaw pain on repeated chewing. Crohn disease **(choice A)** is associated with ankylosing spondylitis and a peripheral arthritis. Dermatomyositis **(choice B)** is associated with polymyositis and polyarthralgias, sometimes with joint swelling and effusions. Discoid lupus erythematosus **(choice C)** is occasionally accompanied by relatively mild arthralgias. Sjögren syndrome **(choice D)** may be accompanied by rheumatoid arthritis-like or lupus-like symptoms.
43
A 71-year-old man with a long history of poorly controlled hypertension presents to the emergency department with headache and visual changes. His blood pressure is 220/130 mm Hg. Current medications include atenolol, nifedipine, thiazide, and clonidine. An IV sodium nitroprusside drip is started. The patient is then transferred to the intensive care unit, where his blood pressure is 135/75 mm Hg; however, he becomes hypoxemic with room air saturations falling to 80%. Which of the following is the most likely reason for this patient’s hypoxemia? (A) Elevation of carboxyhemoglobin levels (B) Elevation of methemoglobin level (C) Hypoventilation (D) Loss of hypoxic pulmonary vasoconstriction (E) Pulmonary embolism
Respuesta: D The **correct answer is D**. Nitroprusside is a nonselective veno- and arteriodilator that works via release of nitric oxide. The intrinsic ability of the lungs to match ventilation with perfusion via vasoconstriction in relatively underventilated lung areas is abolished with this therapy. Therefore, large areas of V/Q mismatch are created that result in often profound hypoxia. Elevation of carboxyhemoglobin levels **(choice A)** is incorrect. Carboxyhemoglobin is the result of binding of carbon monoxide to hemoglobin. This moiety cannot bind oxygen and results in a drastically reduced oxygen-carrying capacity. This patient has not been exposed to carbon monoxide. The byproduct of nitroprusside administration is cyanide. Methemoglobin **(choice B)** is an oxidized form of hemoglobin in which the iron is in the Fe3+ configuration. This often results from blood being exposed to strong oxidizers such as nitrites. Methemoglobinemia is suspected in a cyanotic patient with normal oxygen tension. This form of hemoglobin cannot carry oxygen. Hypoventilation **(choice C)**, which is one of the four mechanisms underlying hypoxemia, is usually quite apparent on clinical inspection. Pulmonary embolism **(choice E)** would certainly produce hypoxia, but in the absence of any clinical or historical evidence for this diagnosis, it is very unlikely at this time.
44
A 45-year-old man with alcoholism is admitted with a diagnosis of acute pancreatitis. He requires large volumes of fluid to maintain blood pressure and urine output, but 24 hours after admission, he appears in stable condition. On the fourth hospital day, the patient develops rapidly progressive respiratory distress, with labored breathing and tachypnea. His temperature is 37.0 C (98.6 F), pulse is 100/min, blood pressure is 128/75 mm Hg, and respirations are 24/min. Intercostal retraction and crackles are appreciated on chest examination. Blood tests show: - Hematocrit: 42% - Leukocytes: 9800/mm3 - Glucose: 110 mg/dL - BUN: 20 mg/dL - AST: 98 U/L - ALT: 60 U/L - Amylase: 280 U/L *Arterial blood gas (room air) - pH 7.32 - Pao2 52 mm Hg - Paco2 51 mm Hg A chest x-ray film reveals diffuse bilateral infiltrates and air bronchograms, a normal cardiac silhouette, and minimal pleural effusions. Which of the following is the most likely diagnosis? (A) Acute bilateral bronchopneumonia (B) Adult respiratory distress syndrome (ARDS) (C) Cardiogenic pulmonary edema (D) Exacerbation of acute pancreatitis (E) Pulmonary embolism
Respuesta: B The **correct answer is B**. The clinical picture is consistent with ARDS, a disorder that may be triggered by a number of different conditions, among which is acute pancreatitis. Typically, ARDS develops 12 to 48 hours following the initiating event (3 to 4 days after acute pancreatitis) and is characterized by acute respiratory failure unresponsive to supplemental oxygen. Therapy includes treatment of the underlying condition and mechanical ventilation with positive end expiratory pressure (PEEP). The overall mortality rate is 50%. Acute bilateral bronchopneumonia **(choice A)** may result in shortness of breath; however, it usually does not cause such severe respiratory failure and is commonly accompanied by productive cough and fever. Furthermore, chest x-ray findings, especially diffuse infiltrates and air bronchograms, are consistent with ARDS and not with bronchopneumonia. Cardiogenic pulmonary edema **(choice C)** must be ruled out because specific treatment is available. A normal cardiac silhouette and the pulmonary changes on chest x-ray do not support a diagnosis of cardiogenic pulmonary edema. In uncertain cases, it may be necessary to measure the pulmonary capillary wedge pressure by Swan-Ganz catheter. Exacerbation of acute pancreatitis **(choice D)**, per se, is excluded by the clinical symptomatology and laboratory findings. There is no hyperglycemia, and leukocytes are only mildly elevated. However, the symptoms of acute pancreatitis can appear to improve while the extrapancreatic complications (i.e., ARDS) worsen. Pulmonary embolism **(choice E)** usually arises from venous thrombosis in the pelvis or in the legs in patients with prolonged immobility or hypercoagulable states. It manifests as acute breathlessness, pleuritic chest pain, tachypnea, and tachycardia. Chest x-ray is often normal or may show oligemia of the affected lung segment, and an arterial blood gas (ABG) study will show hypoxemia and hypocapnia. This patient developed progressive respiratory distress and signs and symptoms of diffuse alveolar lung damage, both of which are inconsistent with acute pulmonary embolism.
45
A 47-year-old woman comes to the physician because she has had several episodes of severe chest pain that awoke her in the early morning. She has no history of major physical illness or drug abuse, drinks alcohol only occasionally, and does not smoke. Her temperature is 37.0 C (98.6 F), blood pressure is 126/78 mm Hg, pulse is 78/min and regular, and respirations are 12/min. An ECG reveals no abnormalities. No further studies are undertaken, and the physician tells the patient that her pain is probably of psychological origin. After a few days, the patient comes to the emergency department at 5 AM complaining of chest pain. An ECG reveals sinus rhythm with ST segment elevation. The patient is admitted, and coronary arteriography is performed, revealing no stenotic lesions. Intravenous administration of ergonovine during arteriography triggers chest pain accompanied by ST elevation on ECG. Which of the following is the most likely diagnosis? (A) Myocardial infarction (B) Prinzmetal angina (C) Psychological chest pain (D) Stable angina (E) Unstable angina
Respuesta: B The **correct answer is B**. The clinical presentation is characteristic of Prinzmetal angina, a form of recurrent myocardial ischemia due to transient coronary vasospasm. ST elevation on ECG during ischemic episodes is highly characteristic. This condition is most often seen in women younger than 50. Coronary angiography frequently fails to disclose any stenotic segments, but the ergonovine test (to be performed with great caution) triggers vasospasm and anginal pain. Myocardial infarction **(choice A)** usually manifests with intense precordial pain that persists for more than 30 minutes associated with characteristic ECG changes. Psychological chest pain **(choice C)** may manifest with variable clinical patterns and is usually associated with other symptoms, such as depression, anxiety, or panic attacks. ECG changes, such as ST elevation, are absent. Precordial pain due to stable angina **(choice D)**, by definition, manifests with a predictable pattern, usually following constant amounts of physical exertion, emotional stress, or exposure to cold temperatures. Unstable angina **(choice E)** usually follows a period of stable angina. The attacks of precordial pain become more frequent and less predictable, and tend to occur at rest. Increasing degrees of coronary artery stenosis and/or platelet thrombi are thought to be the underlying pathologic substrate.
46
A 34-year-old female flight attendant presents with a recurring, sharp pain radiating from her left ear to her mouth. She describes the pain as intense but intermittent, precipitated by cold, light touch, and chewing. Neurologic examination is normal. A tentative diagnosis of trigeminal neuralgia is made, and carbamazepine is prescribed. She returns 6 weeks later complaining of the same pain on both sides of her face and a new onset of urinary incontinence. Which of the following is the most likely diagnosis? (A) Acoustic neuroma (B) Amyotrophic lateral sclerosis (C) Bell palsy (D) Multiple sclerosis (E) Myasthenia gravis
Respuesta: D The **correct answer is D**. The most likely diagnosis in this young woman is multiple sclerosis. Trigeminal neuralgia typically occurs in patients older than 50; however, onset at a young age, bilaterality, and presence of objective signs of sensory loss on the affected side raises the suspicion of multiple sclerosis (MS). MS should be suspected in a patient with multiple neurologic findings that area “separated by time and location.” Acoustic neuroma **(choice A)** is a tumor at the cerebellopontine angle. It is associated with sensory hearing loss, facial nerve palsy, cerebellar dysfunction on the affected side, and headache. It is common in patients with neurofibromatosis. Amyotrophic lateral sclerosis **(choice B)** is a form of progressive motor neuron disease characterized by both upper and lower motor neuron involvement; fasciculations, muscle wasting, and weakness are observed. Even in the late stages of illness, sensory, bowel, bladder, and cognitive functions are preserved. Dementia is unusual. The illness is relentlessly progressive, leading ultimately to death. Bell palsy **(choice C)** is characterized by loss of facial nerve function. Facial paralysis, loss of taste sensation, and hyperacusis may all be produced. Myasthenia gravis **(choice E)** is an autoimmune neuromuscular disorder involving skeletal muscles. The fundamental defect is a decrease in the number of available acetylcholine receptors at the postsynaptic muscle membrane. Sensory changes, bladder and bowel involvement, and loss of tendon reflexes are not usually seen.
47
A 32-year-old woman is brought to the emergency department following the rapid onset of profound malaise and fever. On arrival, the patient’s temperature is 39.7 C (103.5 F), blood pressure is 110/75 mm Hg, pulse is 110/min and regular, and respirations are 17/min. On examination, needle tracks and scars are noted on the forearms and thighs. Chest examination reveals a systolic murmur along the left lower sternal border. Blood tests show 16,000 leukocytes/mm3 and an erythrocyte sedimentation rate of 90/min. Which of the following is the most appropriate next step in management? (A) Broad spectrum antibiotic therapy (B) Echocardiographic studies (C) Three sets of blood cultures (D) Toxicologic studies on blood and urine (E) Ventilation-perfusion lung scans
Respuesta: C The **correct answer is C**. The rapid progression of a febrile illness, the systolic murmur, and the objective evidence indicating intravenous drug abuse point to infective endocarditis as the underlying etiology. The current recommendation is to obtain three different blood samples for culture over a 24-hour period before initiating antibiotic therapy, unless the patient’s condition is critical. This allows identification of the infectious agent and aids in the choice of the most appropriate antibiotic regimen. Intravenous drug abusers are particularly prone to right-sided infective endocarditis, especially Staphylococcus aureus infections on the tricuspid valve. Note that murmurs are infrequent in right-sided infections. Broad spectrum antibiotic therapy **(choice A)** should be started immediately after obtaining the necessary blood cultures. Until culture results are available, the combination of antibiotics should cover the three most frequent organisms responsible for infective endocarditis affecting native valves: viridans streptococci, Staphylococcus aureus, and Enterococcus. Echocardiographic studies **(choice B)** may document the presence of vegetations. Transthoracic echocardiography has a sensitivity of approximately 60%, whereas transesophageal is 90% sensitive in identifying vegetations. Toxicologic studies on blood and urine **(choice D)**, although generally useful to document use of illicit drugs, would not be helpful as a diagnostic tool in this case. Ventilation-perfusion lung scans **(choice E)** are used for the diagnosis of pulmonary infarction, which may occur as a result of embolism from right-sided vegetations.
48
An 18-year-old man comes to clinic for evaluation of weakness and fatigue lasting 6 weeks. Before these past 6 weeks, he reports being fairly healthy. He did, however, have a recent case of “the flu.” On reviewing his medical records, it seems that approximately 2 months ago the patient had a mild hepatitis of unclear etiology (serologies for hepatitis A, B, and C were negative) that has since resolved. Before this illness, he has been healthy, takes no medications, and knows of no diseases that run in his family. He does not use illicit substances, does not smoke, rarely drinks alcohol, has never received a blood transfusion, has never had sex, and does not have any tattoos. Physical examination reveals marked pallor and a 2/6 nonradiating systolic murmur heard best at the right upper sternal border. Abdominal examination reveals a few scattered petechiae but no hepato-splenomegaly. Laboratory studies show: - Hematocrit: 15% - Hemoglobin: 5.0 g/dL - Leukocytes: 4,000/mm3 (normal differential) - Platelets: 15,500/mm3 - Reticulocytes: 0.5% The rest of the patient’s laboratory studies, including a set of chemistries and liver function tests, are unremarkable. The patient is admitted to the hospital and receives a transfusion with leukocyte-reduced blood products. A bone marrow biopsy is performed the next morning that shows cellularity of less than 5% with normal cellular morphology and no organisms on Gram stain. Which of the following is the most appropriate treatment? (A) Antithymocyte globulin (B) Bone marrow transplant (C) Broad-spectrum antibiotics (D) Colony-stimulating factor (E) Intravenous corticosteroids
Respuesta: B The **correct answer is B**. Patients with severe aplastic anemia need a bone marrow transplant. If the patient needs blood products, they should be leukocyte-reduced to reduce allosensitization in likely transplant candidates (even in bone marrow transplant patients). The cause of this patient’s aplastic anemia is not clear. Up to 50% of cases are idiopathic, though many of these may be from an undiagnosed viral infection that induces a host immune response or is directly marrow toxic. A small subset of cases follows an undiagnosed hepatitis, though there is no link with known viral hepatitis subtypes. Immune-modulating drugs, such as antithymocyte globulin **(choice A)**, azathioprine, methotrexate, and corticosteroids **(choice E)** are used in different bone marrow transplant protocols. In older patients or patients without good graft matches, they may be used as definitive therapy. The primary treatment of severe aplastic anemia, however, is bone marrow transplantation. Broad-spectrum antibiotics **(choice C)** are not indicated, as there is no evidence of a bacterial infection causing this patient’s symptoms. Colony-stimulating factors **(choice D)** do not treat aplastic anemia, which is primary marrow failure. Transplantation is necessary.
49
A 70-year-old woman has been in good health until 3 months ago, when she noted the appearance of plaques in her axillae and groin. She has also experienced anorexia and easy fatigability for the past few months. The patient has no history of major disease, and her family history is unremarkable. She is 165 cm tall (65 in) and weighs 53 kg (117 lb). Examination reveals several slightly raised, brown plaques with a velvety surface in the flexural regions of neck, axilla, and groin. A biopsy is consistent with acanthosis nigricans. (A) Allergen skin testing (B) Bacterial cultures of skin scrapings (C) Blood glucose measurement (D) Gluten-free diet trial (E) HIV testing (F) Microscopic examination of KOH-treated skin scrapings (G) Serum IgE assays (RAST or ELISA) (H) Stroking on skin lesions with a blunt object
Respuesta: C The **correct answer is C**. Acanthosis nigricans is a benign skin condition histologically characterized by papillomatous hypertrophy of the epidermis with hyperpigmentation. The lesions involve flexural regions of the body, particularly the axilla and groin. Although the lesions are benign, the sudden appearance of acanthosis nigricans in an elderly woman should raise the suspicion of an underlying malignancy, most commonly gastric cancer. Thus, extensive clinical and radiologic screening must be undertaken to rule out this possibility. Acanthosis nigricans is also associated with endocrinopathies, such as acromegaly, Cushing syndrome, hyperthyroidism, and glucose intolerance. Laboratory tests to screen for such diseases, including fasting glycemia **(choice C)**, are thus appropriate. Glucose intolerance in elderly patients with acanthosis nigricans is due to insulin resistance resulting from autoantibodies to insulin-receptors.
50
A 40-year-old man presents with numerous brown spots on the skin. He reports a history of chronic diarrhea and generalized itching, the latter manifesting frequently after he takes aspirin or drinks alcoholic beverages. He has noticed that slight rubbing on affected skin results in redness and itching. He has had recurrent asthmatic episodes in the past few years. His recent medical history is also significant for 8-kg (18-lb) weight loss in the past 6 months. On examination, there are dozens of small, red-brown macules on the trunk and upper limbs. The spleen is palpable 4 cm below the left costal arch. A biopsy of these macules reveals a dense dermal infiltrate of cells that stain positively for toluidine blue. (A) Allergen skin testing (B) Bacterial cultures of skin scrapings (C) Blood glucose measurement (D) Gluten-free diet trial (E) HIV testing (F) Microscopic examination of KOH-treated skin scrapings (G) Serum IgE assays (RAST or ELISA) (H) Stroking on skin lesions with a blunt object
Respuesta: H The **correct answer is H**. The clinical history alone is highly characteristic of systemic mastocytosis (urticaria pigmentosa), which is a proliferative disorder of mast cells. The infantile type is usually confined to the skin, whereas the adult type affects visceral organs as well. The most telling signs include the brown macules that urticate on stroking or rubbing **(choice H)**, itching triggered by aspirin or alcohol ingestion, asthma, and splenomegaly. Development of a wheal on gentle stroking with a blunt object (for example the handle of the reflex hammer) is a useful diagnostic test. Loss of weight and splenomegaly suggest systemic involvement. The bone marrow is the most commonly involved organ (besides the skin), and x-ray investigations will reveal osteolytic foci. Skin biopsies show accumulation of mast cells in the dermis. Metachromatic staining with toluidine blue is used for visualization of mast cells in tissue sections. Allergen skin testing **(choice A)** and serum IgE assays **(RAST or ELISA; choice G)** are used in the diagnosis of allergic disorders and allow identification of the inciting agents. Neither test is indicated in the diagnosis of urticaria pigmentosa (systemic mastocytosis). Bacterial cultures of skin scrapings **(choice B)** are useful in identifying bacterial pathogens in skin lesions suspected to be of infectious nature. Neither of the above conditions is due to bacterial organisms. Gluten-free diet trial **(choice D)** is a diagnostic and therapeutic approach to cases of suspected dermatitis herpetiformis/celiac disease. Dermatitis herpetiformis manifests with pruritic vesicles on the trunk, which shows characteristic IgA deposition in the tips of dermal papillae, whereas celiac disease is associated with chronic diarrhea and malabsorption. Often, these disorders occur in the same patient and probably result from an abnormal immunologic reaction to gliadin, a protein in the gluten. HIV testing **(choice E)** would be unnecessary in either case. HIV infection is associated with a number of skin conditions, including opportunistic infections (e.g., candidiasis, herpes simplex, herpes zoster, Staphylococcus, and bacillary angiomatosis) and neoplastic conditions (e.g., Kaposi sarcoma and lymphoma). Neither acanthosis nigricans nor mastocytosis has been reported in AIDS. Microscopic examination of KOH-treated skin scrapings **(choice F)** allows identification of fungal organisms in skin lesions.